Bar Prep Flashcards

1
Q

Due on Sale clause

A

A due-on-sale clause allows a lender to demand full payment of the remaining mortgage debt if the debtor transfers the mortgaged property without the lender’s permission. Although labeled a “due on sale” clause, the clause is triggered by most transfers—including gifts.

How well did you know this?
1
Not at all
2
3
4
5
Perfectly
2
Q

Libel

A

Libel is defamation in words written, printed, or otherwise recorded in permanent form. Most courts addressing the issue have held that email messages are categorized as libel. A libel claim requires proof of the following:

The defendant knowingly made a false statement about the plaintiff or negligently failed to determine its falsity.

The statement was defamatory, meaning that the statement would have a general tendency to harm the plaintiff’s reputation.

The defendant intentionally or negligently communicated the statement to a third party.

Once proved, the plaintiff can recover general damages, which are any damages that compensate the plaintiff for harm to the plaintiff’s reputation. Reputational harm can be presumed for libel because it is more permanent and more easily spread than slander (ie, spoken defamation).

How well did you know this?
1
Not at all
2
3
4
5
Perfectly
3
Q

attorney client privilege

A

The attorney-client privilege applies to (1) communications between a client and an attorney that (2) were made to obtain or provide legal assistance for the client and (3) were intended to be and kept confidential. As a result, such communications need not be produced during discovery or at trial.

How well did you know this?
1
Not at all
2
3
4
5
Perfectly
4
Q

breach of contract

A

compensatory damages: expectation (the lost value of the breaching party’s performance) and consequential (losses arising from non breaching party’s special circumstances that were reasonably foreseeable to the breaching party when contract formed or were known by the breaching party

How well did you know this?
1
Not at all
2
3
4
5
Perfectly
5
Q

Methods of discovery

A

Depositions
Written or oral examination of party or witness under oath
10 per party

Interrogatories
Written questions served on party
Serve up to 25
Written responses due within 30 days of service

Request for production
Request served on party (or subpoena served on nonparty) to produce & allow inspection of documents, electronic information, tangible items, or land
No limit
Written response due within 30 days of service

Requests for admission
Requests served on other party to admit truth of facts within scope of discovery
No limit
Written response due within 30 days of service

Physical/mental exams
Order by court or parties’ agreement for physical or mental examination of party if those conditions are in controversy

How well did you know this?
1
Not at all
2
3
4
5
Perfectly
6
Q

Campaign finance restrictions

A

The First Amendment protects political speech, which includes campaign contributions and expenditures. A law that restricts campaign contributions is unconstitutional unless the government satisfies intermediate scrutiny by showing that the law is closely drawn to serve (ie, substantially related to) an important government interest.

How well did you know this?
1
Not at all
2
3
4
5
Perfectly
7
Q

Fed immunity against taxes

A

The federal government, its agencies, and its instrumentalities are immune from direct taxation by the states unless Congress expressly consents.

How well did you know this?
1
Not at all
2
3
4
5
Perfectly
8
Q

Miranda warnings

A

must be given prior to custodial interrogation. drivers are generally not in custody during traffic stops so warning not needed

How well did you know this?
1
Not at all
2
3
4
5
Perfectly
9
Q

battery

A

A prima facie case for battery requires that the plaintiff show that (i) the defendant intended to cause contact with the plaintiff’s person, (ii) affirmative conduct caused such a contact, and (iii) the contact caused bodily harm or was offensive to the plaintiff.

Contact is harmful when it causes injury, physical impairment, pain, or illness.

Contact is offensive when a reasonable person would find the contact offensive.

A defendant intentionally causes the contact if he acts with the desire to bring about the contact or engages in action knowing that the contact is substantially certain to occur.

In most jurisdictions, the defendant need only intend to bring about the contact, not that the defendant intends the contact to be harmful or offensive (often called the single-intent rule).

An employer is vicariously liable for an employee’s torts if the employer has the right to control the activities of the employee.

How well did you know this?
1
Not at all
2
3
4
5
Perfectly
10
Q

consent- battery defense

A

Consent is a defense to battery and can be express, apparent, or presumed. Apparent consent exists and a defendant will not be liable for otherwise tortious conduct if the defendant reasonably believes that the plaintiff actually consents to the conduct, even if the plaintiff does not

How well did you know this?
1
Not at all
2
3
4
5
Perfectly
11
Q

strict liability- manufacturing defect

A

To prevail on a strict liability claim based on a manufacturing defect, the plaintiff must show that (i) the product was defectively manufactured, (ii) the defect existed when the product left the defendant’s control, (iii) the defect caused the plaintiff’s injury, and (iv) the product was used in a reasonably foreseeable way.

How well did you know this?
1
Not at all
2
3
4
5
Perfectly
12
Q

embezzlement

A

when a person: 1) lawfully possesses another’s property (has the property owner’s permission to possess and control it) and 2) fraudulently converts it (substantially interfere with the owner’s rights in property with the specific intent to defraud the owner of that property)

How well did you know this?
1
Not at all
2
3
4
5
Perfectly
13
Q

larceny

A

is the 1) unlawful taking and carrying away of another’s personal property 2) with the specific intent to permanently deprive the owner of that property

How well did you know this?
1
Not at all
2
3
4
5
Perfectly
14
Q

accomplice

A

must aid or encourage the principal with the intent to further the principal criminal aim. such intent can be inferred when the accomplice has a personal stake in the crime being committed

How well did you know this?
1
Not at all
2
3
4
5
Perfectly
15
Q

statements by opposing party

A

excluded from the rule against hearsay, and are admissible as substantive evidence, when offered against that party

How well did you know this?
1
Not at all
2
3
4
5
Perfectly
16
Q

rule against hearsay

A

bars admission of out of court statements offered to prove the truth of the matter asserted. but it does not bar out of court statements offered for different purpose:

to prove statement was made
to ID a statement’s contents, but not to prove its truth, or
to show a statement’s effect on the person who heard it

How well did you know this?
1
Not at all
2
3
4
5
Perfectly
17
Q

contract misrepresentation

A

nondisclosure is equivalent to an assertion that a fact does not exist if the party not disclosing the fact knows that disclosure is necessary to prevent a previous assertion from being fraudulent. basically obligated to correct previous assertions to prevent them from being fraudulent

How well did you know this?
1
Not at all
2
3
4
5
Perfectly
18
Q

venue

A

when suit removed to fed court, venue is proper in fed district of state court where suit was removed even if venue cannot be established under the general venue statute

How well did you know this?
1
Not at all
2
3
4
5
Perfectly
19
Q

joint and several liability

A

default rule on MBE. when multiple Ds engage in tortious conduct (must be proven) and one or more of them causes injury to P, all of the Ds are jointly and severally liable for the harm

How well did you know this?
1
Not at all
2
3
4
5
Perfectly
20
Q

doctrine of alternative liability

A

applies when: 1) multiple tortfeasors 2) at least one of them caused the Ps harm 3) it is impossible to determine which one did it. then burden shifts to Ds to prove each did not cause harm

How well did you know this?
1
Not at all
2
3
4
5
Perfectly
21
Q

bill of attainder clauses

A

prohibit legislative acts that inflict civil or criminal punishment on named individuals, or easily ID groups of individuals, without a trial. exclusion from profession is a type of civil punishment

How well did you know this?
1
Not at all
2
3
4
5
Perfectly
22
Q

ex post facto clauses

A

prohibit enactment of retroactive criminal law

How well did you know this?
1
Not at all
2
3
4
5
Perfectly
23
Q

parol evidence rule

A

bars evidence of prior or contemporaneous communications that supplement or contradict the terms of a contract that is completely integrated. rule does not apply to evidence offered to prove that no contract was formed

How well did you know this?
1
Not at all
2
3
4
5
Perfectly
24
Q

UCC merchants confirmation

A

K between two merchants enforceable against both parties if 1_ one party sends written and signed confirmation, 2) recipient has reason to know contents and 3) recipient fails to object in writing within 10 days

How well did you know this?
1
Not at all
2
3
4
5
Perfectly
25
Q

declarant former testimony

A

admissible if 1) given at trial hearing or deposition in same case or a different proceeding that involved similar parties and issues 2) the party against whom testimony offered had opportunity and similar motive to develop testimony through examination of the declarant, and 3) declarant unavailable

How well did you know this?
1
Not at all
2
3
4
5
Perfectly
26
Q

declarant unavailable when

A

1) asset from the trial or hearing and 2) proponent could not obtain the declarants attendance by subpoena or other reasonable means

How well did you know this?
1
Not at all
2
3
4
5
Perfectly
27
Q

conditional federal funding

A

valid if conditions 1) are clear and unambiguous 2) are reasonably related to the purpose for which the funds are expended 3) do not require recipients to engage in unconstitutional activity, and 4) are not duly coercive

How well did you know this?
1
Not at all
2
3
4
5
Perfectly
28
Q

Federal official sued in official capacity

A

must be served pursuant to rules for serving individual. also process must be served on US gov but if P fails, given reasonable time to cure

How well did you know this?
1
Not at all
2
3
4
5
Perfectly
29
Q

priority of mortgages and other liens

A

generally determined by order of recording unless mortgage enjoys super priority as a purchase money mortgage

How well did you know this?
1
Not at all
2
3
4
5
Perfectly
30
Q

installment land contract

A

under this, time for seller to deliver marketable title arrives when the buyer makes the final installment payment

How well did you know this?
1
Not at all
2
3
4
5
Perfectly
31
Q

murder

A

requires proof D acted with malice aforethought aka 1) intent to kill, inflict serious bodily harm, or commit inherently dangerous felony, or 2) reckless disregard for high or obvious risk of death or serious bodily harm (i.e depraved heart murder)

How well did you know this?
1
Not at all
2
3
4
5
Perfectly
32
Q

doctrine of curative admissibility

A

allows otherwise inadmissible evidence to be admitted to rebut prejudicial evidence when 1) the evidence was improperly admitted through no fault of the prejudiced party, 2) party objected to and moved to strike evidence, and 3) an instruction to disregard cannot remedy the prejudice

How well did you know this?
1
Not at all
2
3
4
5
Perfectly
33
Q

common law oral modification

A

CL ignores “no oral modification” clauses and allows written K to be modified orally unless modified K falls under statute of frauds. under UCC if two merchants with NOM clause then always enforceable. if one merchant then NOM clause in merchants form must be signed by non merchant to be enforceable

How well did you know this?
1
Not at all
2
3
4
5
Perfectly
34
Q

supremacy clause

A

bars state courts from discriminating against cases arising under federal law. as a result, state courts can only refuse to hear federal law claims if 1) congress expressly granted federal courts exclusive jurisdiction or 2) neutral state rule prohibits the state court from hearing the case

How well did you know this?
1
Not at all
2
3
4
5
Perfectly
35
Q

products liability based on misrepresentation

A

arises when 1) the D (commercial supplier) misrepresents a material fact about producer 2) consumer justifiably relied on misrepresentation and 3) product caused the plaintiff harm

How well did you know this?
1
Not at all
2
3
4
5
Perfectly
36
Q

article III gives US Supreme Court

A

original jurisdiction over cases that 1) affect ambassador public minister or consul or 2) in which state is a party. Congress cannot alter original jurisdiction

How well did you know this?
1
Not at all
2
3
4
5
Perfectly
37
Q

search incident to lawful arrest

A

police dont need warrant if search is limited to 1) person arrested and 2) areas within his or her reach

How well did you know this?
1
Not at all
2
3
4
5
Perfectly
38
Q

expert testimony

A

admissible if 1) relevant, helps trier of fact understand evidence or fact and 2) reliable, based on sufficient facts or date and product of reliable principles

How well did you know this?
1
Not at all
2
3
4
5
Perfectly
39
Q

race notice

A

Bona fide purchaser interest will have priority if BFP: lacked notice of earlier interest at the time BFD acquired interest AND recorded interest before the earlier interest was recorded

How well did you know this?
1
Not at all
2
3
4
5
Perfectly
40
Q

supplemental jurisdiction

A

fed court may exercise over a claim outside of the court’s original SMJ if it shares a common nucleus of operative facts with an original jurisdiction claim

How well did you know this?
1
Not at all
2
3
4
5
Perfectly
41
Q

request for admission

A

party served with request must then serve a written response within 30 days that:

admits matter
specifically denies matter
explains why matter cannot be admitted nor denied
objects that the matters falls outside the scope of discovery

How well did you know this?
1
Not at all
2
3
4
5
Perfectly
42
Q

invasion of privacy based on public disclosure of private facts

A

defendant privileged to disclose when 1) was in a fair and accurate report of public meeting and 2) meting dealt with matters of legit public interest (newsworthy)

How well did you know this?
1
Not at all
2
3
4
5
Perfectly
43
Q

strict products liability exception

A

someone who merely used a defective product to provide a service is not subject to strict products liability for harm to persons or property caused by the defect

How well did you know this?
1
Not at all
2
3
4
5
Perfectly
44
Q

waiving service of process

A

P must send via 1st class mail or other reliable means. must name the court and date the request was sent. must contain copy of complaint, two copies of waiver form, and prepaid means for returning signed waiver.

How well did you know this?
1
Not at all
2
3
4
5
Perfectly
45
Q

remainders

A

vested and contingent free transferable during grantee’s lifetime, devisable by will, and inheritable through intestate succession.

vested- not subject to any condition precedent AND held by living ID person (remainder to by youngest son”

contingent- subject to condition precedent OR held by unknown or unborn person (“remainder to my youngest son but only if”)

How well did you know this?
1
Not at all
2
3
4
5
Perfectly
46
Q

article III fed court standing

A

only to determine merits of actual cases or controversies

How well did you know this?
1
Not at all
2
3
4
5
Perfectly
47
Q

larceny intent

A

to permanently deprive another of his/her property

How well did you know this?
1
Not at all
2
3
4
5
Perfectly
48
Q

fraud (intentional misrep or deceit) prima facie elements

A

defendant knowingly or recklessly misrepresented a material fact with the intent to induce the plaintiffs reliance and the plaintiff justifiably relied on the misrepresentation and suffering pecuniary (financial loss) as a result.

reliance not justifiable if the rep was obviously false or the defendant was stating law opinion

How well did you know this?
1
Not at all
2
3
4
5
Perfectly
49
Q

grand jury

A

exclusionary rule does not apply during grand jury proceeding since the grand jury is an investigatory body not a judicial one with the power to convict

How well did you know this?
1
Not at all
2
3
4
5
Perfectly
50
Q

seller’s duty to disclose

A

common law (minority) - no duty to disclose property defects unless otherwise provided

modern (majority)- no duty for commercial property. must disclose known material defects that buyer could not reasonably discover for residential property unless “as is” clause without seller fraud

defect = material if: 1) substantially effects value of the residence, 2) impacts the health or safety of a resident, or 3) affects the desirability of the residence to the buyer

How well did you know this?
1
Not at all
2
3
4
5
Perfectly
51
Q

excited utterances

A

excepted from rule against hearsay. statements related to a startling event or condition that are made while the declarant is still under the stress of excitement from the event or condition

How well did you know this?
1
Not at all
2
3
4
5
Perfectly
52
Q

present sense impressions

A

excepted from rule against hearsay. statements describing or explaining an event or condition that are made while or immediately after the declarant perceived it

How well did you know this?
1
Not at all
2
3
4
5
Perfectly
53
Q

burglary (common law)

A

unlawful breaking and entering of another’s dwelling at night with the specific intent to commit a felony (usually larceny)

How well did you know this?
1
Not at all
2
3
4
5
Perfectly
54
Q

negligent misrep

A

only for commercial setting when 1) D provides P with false info negligently and 2) P justifiably relies on info and suffers pecuniary (financial loss)

How well did you know this?
1
Not at all
2
3
4
5
Perfectly
55
Q

rational basis

A

default test. challenger has burden to prove that the government action is not rationally related to a legitimate government interest

How well did you know this?
1
Not at all
2
3
4
5
Perfectly
56
Q

intermediate scrutiny

A

quasi suspect class. government has burden to prove its action is substantially related to an important government interest

How well did you know this?
1
Not at all
2
3
4
5
Perfectly
57
Q

strict sructiny

A

fundamental right or suspect class. government has burden to prove that its action is necessary to serve a compelling government interest

How well did you know this?
1
Not at all
2
3
4
5
Perfectly
58
Q

unincorporated association

A

such as partnership. is a citizen of every state where its members are domiciled

How well did you know this?
1
Not at all
2
3
4
5
Perfectly
59
Q

trade fixtures

A

items attached to real property by tenant for use in trade or business can be removed without landlord consent if 1) removal occurs before, or within reasonable time after, lease ends and 2) removal will not substantially harm the property

How well did you know this?
1
Not at all
2
3
4
5
Perfectly
60
Q

accessions

A

items so attached to property that they cannot be removed without causing substantial damage

How well did you know this?
1
Not at all
2
3
4
5
Perfectly
61
Q

permissible race based affirmative action

A

minority set adds for gov employment/contracts- remedying its own history of racial discrimination

race based students assignments in public schools - remedying past intentional racial segregation in public schools

race based admissions policies - achieving diverse student body in higher education

government interest on the right of the hyphen

How well did you know this?
1
Not at all
2
3
4
5
Perfectly
62
Q

subject matter jurisdiction

A

federal question jurisdiction- case arises under constitution, a federal law, or treaty

diversity jurisdiction- amount in controversy >75l and parties are citizens of different states

How well did you know this?
1
Not at all
2
3
4
5
Perfectly
63
Q

criminal acts- superseding cause

A

generally superseding cause unless the defendant had reason to know that the negligence would increase the risk that such acts would occur

How well did you know this?
1
Not at all
2
3
4
5
Perfectly
64
Q

full faith and credit clause

A

requires states to respect the public acts, records, and judicial proceedings of other states. under this clause, a state court must recognize and enforce judicial decisions rendered by a court in another state when:
1) court has personal and subject matter jurisdiction

How well did you know this?
1
Not at all
2
3
4
5
Perfectly
65
Q

full faith and credit clause

A

requires states to respect the public acts, records, and judicial proceedings of other states. under this clause, a state court must recognize and enforce judicial decisions rendered by a court in another state when:
1) court has personal and subject matter jurisdiction
2) case was decided on its merits
3) a final judgment was entered

How well did you know this?
1
Not at all
2
3
4
5
Perfectly
66
Q

intentional infliction of emotional distress theories

A

direct- P was direct victim of D conduct
bystander- P contemporaneously perceived the D’s extreme and outrageous conduct

How well did you know this?
1
Not at all
2
3
4
5
Perfectly
67
Q

negligent hiring liability

A

arises when D’s failure to use reasonable care in hiring a competent worker causes the plaintiff harm- there must be some causal connection between the D’s negligence in hiring the worker, the worker’s employment or job duties, and the plaintiff’s harm

How well did you know this?
1
Not at all
2
3
4
5
Perfectly
68
Q

statements adopted by opposing party

A

excluded from rule against hearsay if they are offered against that party. includes statements made by another that an opposing party explicitly (verbal agreement) or implicitly (conduct or silence). Adoption by silence occurs when: the silent party understood and had the ability to deny the statement AND a reasonable person would have denied the statement under similar circumstances if it was untrue

How well did you know this?
1
Not at all
2
3
4
5
Perfectly
69
Q

joint and several liability

A

arises when 2 or more Ds were negligent and any one of them could have caused the Ps indivisible har. This allows plaintiff to recover the full amount from any of the negligent Ds even if it is impossible to prove which one actually caused it. However, the P must first prove that each D was negligent.

burden shifts to each D to prove negligence didn’t cause P harm only after the P proves that each D was negligent to begin with

How well did you know this?
1
Not at all
2
3
4
5
Perfectly
70
Q

content-based 1A speech regulation

A

strict scrutiny applies and law is unconstitutional unless the gov can show that it is necessary and narrowly tailored (i.e least restive means) to achieve a compelling government interest

How well did you know this?
1
Not at all
2
3
4
5
Perfectly
71
Q

content-based 1A speech regulation

A

strict scrutiny applies and law is unconstitutional unless the gov can show that it is necessary and narrowly tailored (i.e least restive means) to achieve a compelling government interest

How well did you know this?
1
Not at all
2
3
4
5
Perfectly
72
Q

content-neutral 1A speech regulation

A

intermediate scrutiny applies and the law is permissible so long as the government can show that it is narrowly tailored to achieve an important government interest and leaves open alternative channels or communciation

How well did you know this?
1
Not at all
2
3
4
5
Perfectly
73
Q

SMJ under diversity

A

court cannot exercise supplemental jurisdiction over another claim that would contaminate diversity of citizenship

How well did you know this?
1
Not at all
2
3
4
5
Perfectly
74
Q

motion to remand

A

return case to state court. one basis for remand is the home court advantage rule ( forum-defendant rule) which prohibits removal when 1) SMJ arises from diversity jurisdiction and 2) a defendant is a citizen of the state where the case was filed

How well did you know this?
1
Not at all
2
3
4
5
Perfectly
75
Q

defense of lack of personal jurisdiction

A

waived if not asserted in a pre-answer motion or an answer, whichever comes first

How well did you know this?
1
Not at all
2
3
4
5
Perfectly
76
Q

personal jurisdiction- general jurisdiction

A

D has continuous and systemic contacts with the forum state that are so substantial that D is essentially at home

How well did you know this?
1
Not at all
2
3
4
5
Perfectly
77
Q

personal JX- specific JX

A

when P claim arises from D’s minimum contacts with the forum state and the exercise of jurisdiction would comply with the notions of fair play and substantial justice

How well did you know this?
1
Not at all
2
3
4
5
Perfectly
78
Q

service of process

A

P must provide proof to court but failure to prove service of process does not affect validity of service

How well did you know this?
1
Not at all
2
3
4
5
Perfectly
79
Q

venue

A

proper in any district where:

any D resides

a substantial part of the events that gave rise to the suit occurred (suit-based venue) or a substantial part of the property at issue is located (property based venue)

any D is subject to the court’s personal jurisdiction- but only if neither of the above apply (fall back provision)

How well did you know this?
1
Not at all
2
3
4
5
Perfectly
80
Q

Contents of answer

A

must serve answer on plaintiff and it must contain:

1) admissions and denials
2) motions that have not been waived
3) affirmative defenses and
4) compulsory counterclaims

otherwise these items will be waived

How well did you know this?
1
Not at all
2
3
4
5
Perfectly
81
Q

deposition

A

discovery method in which a party conducts a written or oral examination of a party or nonparty under oath and outside of court

How well did you know this?
1
Not at all
2
3
4
5
Perfectly
82
Q

judgment has a matter of law JMOL

A

a party can move for it at any time before the case is submitted to the jury. District court must:

1) view evidence and draw all reasonable inferences in light more favorable to the nonmovant
2) disregard any evidence favorable to the movant that the jury is not required to believe and
3) not consider the credibility of witnesses or evaluate the weight of evidence

How well did you know this?
1
Not at all
2
3
4
5
Perfectly
83
Q

jury verdict

A

must be unanimous and returned by a jury of at least 6 (but not more than 12) members unless the parties stipulate otherwise

How well did you know this?
1
Not at all
2
3
4
5
Perfectly
84
Q

abuse of discretion review

A

standard of appellate review for trial court’s discretionary rulings, such as whether to admit evidence. under this standard, only a clearly arbitrary or unreasonable ruling will be reversed

How well did you know this?
1
Not at all
2
3
4
5
Perfectly
85
Q

collateral estoppel

A

Collateral estoppel

(ie, issue preclusion)

Mutual v. Nonmutual

Mutual

Parties from first action assert collateral estoppel in subsequent action against other parties from first action

Nonmutual

Nonparties from first action assert collateral estoppel in subsequent action against parties from first action

Defensive v. Offensive

Defensive

Defendant in second action asserts to avoid relitigating issue from first action

Offensive

Plaintiff in second action asserts to establish issue from first action. Not permitted if:

plaintiff could have easily joined first action
defendant had little incentive to vigorously defend in first action
second action affords procedural opportunities unavailable in first action or
inconsistent findings on issue exist
Collateral estoppel (i.e., issue preclusion) precludes the relitigation of an issue that was actually litigated, determined, and essential to a valid final judgment on the merits. There are two types of collateral estoppel:

Mutual – where parties from the first action assert collateral estoppel in a subsequent action against other parties from the first action

Nonmutual – where nonparties from the first action assert collateral estoppel in a subsequent action against parties from the first action

Both types are permitted in federal court, and both can be used in a defensive or offensive manner. Defensive estoppel is used by a defendant in the second action to avoid relitigating an issue from the first action. Offensive estoppel is used by a plaintiff in the second action to establish an issue from the first action—with limited exceptions (see table above).

How well did you know this?
1
Not at all
2
3
4
5
Perfectly
86
Q

judge presiding unable to proceed

A

another judge may take over so long a this successor judge: certifies that he/she is familiar with the record and determines that the case may be completed without prejudice to the parties. if not, a new trial will be ordered

How well did you know this?
1
Not at all
2
3
4
5
Perfectly
87
Q

dormant commerce clause

A

negative implication of commerce clause that prohibits states from discriminating against or otherwise unduly burdening interstate commerce. states are not restrained by this clause when they participate in the market, for example, buying or selling goods or services.

How well did you know this?
1
Not at all
2
3
4
5
Perfectly
88
Q

refreshing recollection

A

any item may be used to refresh witness’s memory regardless of admissibility if:
1) witness once knew but is now unable to recall fact or event AND
2) item will hep witness recall that information

How well did you know this?
1
Not at all
2
3
4
5
Perfectly
89
Q

recorded recollection

A

record is admissible hearsay if it:
1) contains info witness once knew but cannot recall well enough to testify fully and accurately
2) was made or adopted by witness when matter was fresh in his/her mind AND
3) accurately reflects witness’s knowledge at time record was made

How well did you know this?
1
Not at all
2
3
4
5
Perfectly
90
Q

intentional infliction of emotional distress

A

P must prove D:
1) in extreme and outrageous manner
2) intended to cause the P severe emotional distress or recklessly disregarded risk of causing such distress AND
3) caused the P severe emotional distress

ex. abusing a position of authority going far beyond what is necessary to exercise that authority

How well did you know this?
1
Not at all
2
3
4
5
Perfectly
91
Q

quantum meruit

A

entitles party who conferred benefit on another in a quasi contractual relationship to damages for the reasonable value of that benefit

How well did you know this?
1
Not at all
2
3
4
5
Perfectly
92
Q

material breach

A

a party in material breach cannot sue to recover contract damages. also, it discharges the other party’s duty to perform and entitles that party to terminate the contract if breaching party is unwilling or unable to cure breach

How well did you know this?
1
Not at all
2
3
4
5
Perfectly
93
Q

Negligent Infliction of emotional distress

A

three theories:
1) zone of danger
2) bystander
3) special situations

How well did you know this?
1
Not at all
2
3
4
5
Perfectly
94
Q

automobile except to warrant requirement

A

allows police to conduct warrantless search of a vehicle if they have probable cause to believe it contains evidence of a crime, they can search any area within vehicle where the evidence might be located including locked containers

How well did you know this?
1
Not at all
2
3
4
5
Perfectly
95
Q

self defense- use of force

A

justified against another if actor 1) actually and reasonably believed the force was necessary to protect against imminent lawful harm, 2) used reasonable force to prevent such harm, and 3) was not initial aggressor

How well did you know this?
1
Not at all
2
3
4
5
Perfectly
96
Q

criminal battery

A

when person unlawfully applies force resulting in bodily harm or offensive contact

How well did you know this?
1
Not at all
2
3
4
5
Perfectly
97
Q

non deadly force - self defense

A

actual and reasonable belief of imminent unlawful harm.
force reasonably necessary to prevent harm.
not initial aggressor.

no duty to retreat

How well did you know this?
1
Not at all
2
3
4
5
Perfectly
98
Q

deadly force- self defense

A

actual and reasonable belief of imminent serious harm or death.
deadly force necessary to prevent harm.
not initial aggressor.

majority rule- no duty to retreat
minority rule- duty when safe retreat available unless inside one’s home

How well did you know this?
1
Not at all
2
3
4
5
Perfectly
99
Q

license for contract enforcement

A

when party does not have required license, enforceability of contract depends on whether the purpose behind the requirement if 1) economic, which renders contract enforceable, or 2) regulatory which means contract is unenforceable if the public policy behind the requirement outweighs the interest in enforcement

How well did you know this?
1
Not at all
2
3
4
5
Perfectly
100
Q

21st amendment

A

grants states broad authority to regulate alcohol within borders but when doing so, states cannot violate the dormant commerce clause by discriminating against interstate commerce unless 1) the state statute furthers a legitimate noneconomic state interest and 2) no reasonable alternative exists to do so

How well did you know this?
1
Not at all
2
3
4
5
Perfectly
101
Q

bailement

A

occurs when one party (bailor) temporarily entrusts his/her personal property to another (bailee) without transferring ownership. Bailee duty of care for the property depends:

bailor receives sole benefit- bailee has a lesser duty to care for property and is liable for harm to property only if bailee has been grossly negligent

bailee receives the sole benefit- must exercise extraordinary care for pretty and liable for harm even if only slightly negligent

both benefit- bailee must take reasonable care of property and is liable under ordinary negligence

How well did you know this?
1
Not at all
2
3
4
5
Perfectly
102
Q

pertinent trait evidence for crim def character

A

Defendant may offer evidence of pertinent character trait by: reputation or opinion testimony

Prosecution may rebut with:reputation or opinion testimony orSIC (only on cross-examination)

How well did you know this?
1
Not at all
2
3
4
5
Perfectly
103
Q

truthful character

A

If defendant testifies, prosecution may impeach with:

reputation or opinion testimony
SIC (only on cross-examination) or
extrinsic evidence of conviction for felony or crime of dishonesty

Defendant may rebut with:

reputation or opinion testimony or
SIC (only on cross-examination)

How well did you know this?
1
Not at all
2
3
4
5
Perfectly
104
Q

congressional vetoes

A

always invalid

How well did you know this?
1
Not at all
2
3
4
5
Perfectly
105
Q

foreclosure methods

A

judicial sale- judicially supervised public sale or mortgaged property

non judicial- privately conducted public sale of mortgaged property (permitted only if mortgage contains power of sale clause) but court cn overturn if auction or sale processes violate due process and purchase price was grossly inadequate (20% market value)

strict foreclosure- foreclosure without sale of mortgaged property

How well did you know this?
1
Not at all
2
3
4
5
Perfectly
106
Q

nonhearsay- out of court statements offered against party opponent

A

that were:

made or adopted by party opponent

made by a person authorized by party opponent to make statement on subject

made by the party opponent’s agent or employee about a matter within scope of that agency or employment

made by the party opponent’s coconspirator during and in furtherance of the conspiracy

How well did you know this?
1
Not at all
2
3
4
5
Perfectly
107
Q

attorney client privilege

A

extends to confidential communication between a client and attorney if those communications are made in furtherance of legal representation

How well did you know this?
1
Not at all
2
3
4
5
Perfectly
108
Q

spousal-communications privilege

A

protects confidential communications during the marriage from disclosure in a civil or criminal case.

How well did you know this?
1
Not at all
2
3
4
5
Perfectly
109
Q

spousal-immunity privilege

A

protects persons from being required to testify against their spouse in a criminal case about events that occurred before or during the marriage.

How well did you know this?
1
Not at all
2
3
4
5
Perfectly
110
Q

discovery of expert materials

A

non testifying expert- Facts known or opinions held by expert not discoverable unless:

relate to court-ordered physical/mental examination
or
exceptional circumstances make it impracticable to obtain information by other means

testifying expert:

Facts known & opinions held by expert discoverable through deposition

Drafts of expert reports & disclosures not discoverable

Communications between attorney & expert not discoverable unless:

relate to expert’s compensation or
identify facts, data, or
assumptions provided by attorney that expert considered in forming opinions

How well did you know this?
1
Not at all
2
3
4
5
Perfectly
111
Q

doctrine of equitable conversion

A

places the risk of loss on the party with equitable title to the property. the buyer acquires equitable title once a real estate contract is formed and can be specifically enforced

How well did you know this?
1
Not at all
2
3
4
5
Perfectly
112
Q

then-existing state of mind

A

excepted from rule against hearsay (statements of motive, intent, plan

How well did you know this?
1
Not at all
2
3
4
5
Perfectly
113
Q

doctrine of estoppel by deed (ie, after-acquired title)

A

effectuates a conveyance of unowned property by warranty deed when the grantor later acquires ownership of that property. The grantor’s ownership interest automatically transfers to the grantee, who can then accept or reject the transfer.

How well did you know this?
1
Not at all
2
3
4
5
Perfectly
114
Q

state action doctrine

A

private entity bound by constitution when the government is so pervasively intertwined in its control or management that the private entity is essentially a government actor

How well did you know this?
1
Not at all
2
3
4
5
Perfectly
115
Q

state taxation of interstate commerce

A

Under the dormant commerce clause, states cannot tax interstate commerce unless the tax is (1) levied on persons or activities that have a substantial nexus with the state, (2) fairly apportioned, (3) nondiscriminatory, and (4) fairly related to state-provided services or benefits.

Under the Tenth Amendment, any power that the Constitution does not expressly grant to the federal government—including the power to construct a state tax system—is reserved to the states. this power is not plenary (ie, absolute).

How well did you know this?
1
Not at all
2
3
4
5
Perfectly
116
Q

land owner traditional duty to unknown or unanticipated trespassers

A

no duty to unknown or unanticipated trespassers. limited duty to known or anticipated trespassers to:

warn about hidden artificial dangers that are known but unlikely to be discovered by trespasser

use reasonable care while conducting activities on land

How well did you know this?
1
Not at all
2
3
4
5
Perfectly
117
Q

fifth amendment double jeopardy

A

prohibits 1) multiple punishments for the same offense and 2) a second prosecution for the same offense following a conviction of an acquittal. prohibition only applies after jeopardy attaches:

in a jury trial when the jury is impaneled and sworn OR

in a bench trial when the judge begins to hear evidence

How well did you know this?
1
Not at all
2
3
4
5
Perfectly
118
Q

statement in learned treatise hearsay exception

A

excepted when 1) publication is established as reasonably reliable authority by a party’s expert or judicial notice and 2) the statements are called to the attention of or relied on by an expert witness during examination.

can be read into evidence, it cannot be received as an exhibit

How well did you know this?
1
Not at all
2
3
4
5
Perfectly
119
Q

preexisting duty rule

A

traditional CL- promise to perform, or performance of, preexisting duty does not constitute consideration

modern exception- consideration exists if preexisting duty owed to third person (non party to contract)

How well did you know this?
1
Not at all
2
3
4
5
Perfectly
120
Q

murder

A

unlawful killing of another committed with some type of malice aforethought

How well did you know this?
1
Not at all
2
3
4
5
Perfectly
121
Q

malice aforethought

A

intent to kill- conscious desire to cause death or substantial certainty that death will result

intent to inflict serious bodily harm- conscious desire to cause serious physical injury or substantial certainty that such injury will result

depraved heart murder- reckless disregard for high risk of death or serious bodily harm

felony murder- killing during an inherently dangerous felony
(BARKK burglary arson rape robbery kidnapping)

How well did you know this?
1
Not at all
2
3
4
5
Perfectly
122
Q

class action settlement

A

must be approved by court. only after court holds hearing and issues findings that the settlement is fair reasonable and adequate.

in common question class actions, court may give members second opportunity to opt out. in these, common questions of law or fact predominate over individual questions and a class action is the best method to fairly and adequately adjustive dispute

How well did you know this?
1
Not at all
2
3
4
5
Perfectly
123
Q

fed diversity jurisdiction not allowed in:

A

probate matters and domestic relations matters

How well did you know this?
1
Not at all
2
3
4
5
Perfectly
124
Q

100 mile bulge provision

A

allows fed courts to acquire PJ over party who is:

added to the suit through imp leader (third party practice) or required joinder AND

served with a process within 100 mules of the federal court where the suit is pending, even if served in another state

How well did you know this?
1
Not at all
2
3
4
5
Perfectly
125
Q

impleader

A

defendant brings in another party that may be liable to the defendant

How well did you know this?
1
Not at all
2
3
4
5
Perfectly
126
Q

special rule for venue- foreign resident

A

can be sued in any judicial district

How well did you know this?
1
Not at all
2
3
4
5
Perfectly
127
Q

ways to add parties to suit

A

required joinder

permissive joinder

intervention

impleader

interpleader

class action

How well did you know this?
1
Not at all
2
3
4
5
Perfectly
128
Q

required joinder

A

requires addition of necessary party to suit

How well did you know this?
1
Not at all
2
3
4
5
Perfectly
129
Q

permissive joinder

A

allows addition of nonessential party to suit

How well did you know this?
1
Not at all
2
3
4
5
Perfectly
130
Q

intervention

A

allows nonparty whose interests may be affected to join suit

How well did you know this?
1
Not at all
2
3
4
5
Perfectly
131
Q

impleader

A

allows D to add nonparty who may be liable to D for all or part of the asserted claim to suit

How well did you know this?
1
Not at all
2
3
4
5
Perfectly
132
Q

interpleader

A

allows possessor of property to force persons who claim ownership of property to resolve dispute in single suit

How well did you know this?
1
Not at all
2
3
4
5
Perfectly
133
Q

class action

A

allows party to represent interests of entire class of similarly situation individuals

How well did you know this?
1
Not at all
2
3
4
5
Perfectly
134
Q

jury polling

A

a court must, on party’s request, or on own initiative, poll the jurors individually after a verdict is returned but before the jury is discharged. if poll reveals verdict is not unanimous, the court can direct the jury to deliberate further or order a new trial

How well did you know this?
1
Not at all
2
3
4
5
Perfectly
135
Q

renewed JMOL

A

can be renewed within 28 days after the entry of judgment to seek to overturn an adverse verdict

How well did you know this?
1
Not at all
2
3
4
5
Perfectly
136
Q

jury size

A

at least 6 but no more than 12

How well did you know this?
1
Not at all
2
3
4
5
Perfectly
137
Q

in diversity actions

A

state law controls substantive issues and federal law controls procedural issues such as jury size

How well did you know this?
1
Not at all
2
3
4
5
Perfectly
138
Q

federal question jurisdiction

A

apply both federal substantive AND procedural law. including federal common law

How well did you know this?
1
Not at all
2
3
4
5
Perfectly
139
Q

fed court personal jursidiction

A

same PJ as the courts of the state it is located in (forum state). forum state’s long arm statute specifics when a fed court within the state can exercise PJ over nonresident defendant

How well did you know this?
1
Not at all
2
3
4
5
Perfectly
140
Q

fed court injunction

A

when 1) movant likely to succeed on merits
2) movant likely to suffer irreparable harm in absence of relief
3) balance of equities in movant favor
4) injunction in best interests of public

if monetary damages can adequately compensate, no irreparable harm

How well did you know this?
1
Not at all
2
3
4
5
Perfectly
141
Q

interrogatories

A

one party serves written questions upon another party- not a nonparty (e.g. witness)

must be a party to the case

How well did you know this?
1
Not at all
2
3
4
5
Perfectly
142
Q

jury trial demand

A

by 1) serving other parties with a written jury trial demand no later than 14 days after the last pleading directed to that issue is served and 2) filing the jury demand with the court within a reasonable time after service of the demand

How well did you know this?
1
Not at all
2
3
4
5
Perfectly
143
Q

due process

A

reasonably apprise D or lawsuit. by most reasonable means.

How well did you know this?
1
Not at all
2
3
4
5
Perfectly
144
Q

oral deposition without court’s leave or parties’ stipulation

A

unless:

1) deposition exceeds the 10- deposition limit
2) deposition is sought before the initial planning conference or
3) the deponent was already deposed in the case

How well did you know this?
1
Not at all
2
3
4
5
Perfectly
145
Q

class action certification

A

numerosity- >40 members so numerous that joining all is impracticable

commonality- class shares common questions of law or fact

typicality- named plaintiff’s claims are typical of the claim of the class

adequacy- named plaintiffs will fairly and adequately protect interests of class

How well did you know this?
1
Not at all
2
3
4
5
Perfectly
146
Q

final judgment rule

A

federal appellate court no JX to hear appeal until district court has issued a final judgment- decision that fully resolves the dispute on the merits. interlocutory appeals statute provides some exceptions to this rule, allowing immediate appeal of certain orders

How well did you know this?
1
Not at all
2
3
4
5
Perfectly
147
Q

fed court following state substantive law

A

must follow rulings issued by the state’s highest court

How well did you know this?
1
Not at all
2
3
4
5
Perfectly
148
Q

counterclaim

A

must be asserted in the D’s answer if:

1) arises from same transaction or occurrence AND
2) does not require adding parties over whom the court cannot acquire JX

failure to do so generally means counterclaim is waived and barred from being raised in subsequent lawsuits. BUT when action dismissed before D’s answer is filed, any counterclaims that would have been compulsory are not waived because the D never had the opportunity to raise them

How well did you know this?
1
Not at all
2
3
4
5
Perfectly
149
Q

class action fairness act

A
  • 100 members at least
  • minimal diversity
  • amount in controversy >5m when claims aggregated
How well did you know this?
1
Not at all
2
3
4
5
Perfectly
150
Q

response to amended pleading

A

unless court orders otherwise, responding party generally must respond to amended pleading within 1) the time that remains to respond to original pleading or 2) 14 days after service of amended pleading- whichever occurs later

How well did you know this?
1
Not at all
2
3
4
5
Perfectly
151
Q

jury instruction error

A

preserved for appeal when a party timely objects to the instruction on the record and states the grounds for objection. objection timely if made promptly after learning jury instruction has been or will be given OR that a request has been refused

How well did you know this?
1
Not at all
2
3
4
5
Perfectly
152
Q

correcting mistake

A

before appeal docketed, a district court can correct a mistake in judgment, order, or other part of the record on its own initiative or pursuant to party’s motion. after an appeal has been docketed, district court can only correct with the appellate court’s leave

How well did you know this?
1
Not at all
2
3
4
5
Perfectly
153
Q

extraordinary relief

A

within one year of the entry of final judgment based on 1) mistake, inadvertence, surprise, excusable neglect, 2) newly discovered evidence, or 3) an opposing party’s fraud, misrepresentation, or misconduct

How well did you know this?
1
Not at all
2
3
4
5
Perfectly
154
Q

new trial

A

sua sponta (court on its own) or via party’s motion within 28 days after final judgment

How well did you know this?
1
Not at all
2
3
4
5
Perfectly
155
Q

failure to deny damages-related allegations

A

does not deem them admitted and conclusively established

How well did you know this?
1
Not at all
2
3
4
5
Perfectly
156
Q

present recollection refreshed

A

witness looks at document notes etc, puts notes aside, and proceeds to testify. document does not become evidence and witness does not read from it

How well did you know this?
1
Not at all
2
3
4
5
Perfectly
157
Q

past recollection recorded

A

when witness still cannot remember. permits witness to read notes into evidence under hearsay exception. requires: that witness cannot remember, record was made when it was fresh in their memory, and the record accurately reflects memory

How well did you know this?
1
Not at all
2
3
4
5
Perfectly
158
Q

cross examination

A

scope limited to subject of direct examination but courts have discretion to allow more inquiry. allowed to use leading questions!

How well did you know this?
1
Not at all
2
3
4
5
Perfectly
159
Q

improper questions

A

compound question, leading questions (with exceptions), facts not in evidence, argumentative questions, questions calling for inappropriate conclusions, repetitive questions

How well did you know this?
1
Not at all
2
3
4
5
Perfectly
160
Q

burden of proof

A

two in one:

burden of production- party must produce enough evidence to get it in front of jury

burden of persuasion- party must convince jury to decide case in its favor

How well did you know this?
1
Not at all
2
3
4
5
Perfectly
161
Q

civil case

A

preponderance of evidence

How well did you know this?
1
Not at all
2
3
4
5
Perfectly
162
Q

criminal case

A

beyond a reasonable doubt standard (higher than civil)

How well did you know this?
1
Not at all
2
3
4
5
Perfectly
163
Q

rebuttable presumption

A

shifts burden of production but not burden of persuasion

How well did you know this?
1
Not at all
2
3
4
5
Perfectly
164
Q

character evidence by defendant in criminal case

A

if D presents reputation or opinion testimony about pertinent trait, prosecutors can rebut and can also rebut by cross examining the defendant’s character witnesses with question about specific acts from the past

How well did you know this?
1
Not at all
2
3
4
5
Perfectly
165
Q

Prior Acts- things not implicated by character evidence rule because different uses of prior acts because don’t implicate propensity

A

M- motive
I- intent
M- absence of mistake
I- identity
C- Common plan or scheme

How well did you know this?
1
Not at all
2
3
4
5
Perfectly
166
Q

impeachment of witness

A

bias

sensory competence

character for truthfulness or untruthfulness

How well did you know this?
1
Not at all
2
3
4
5
Perfectly
167
Q

NOT hearsay

A

Certain prior statements of testifying witnesses
1) prior inconsistent statements
2) prior consistent statements
3) prior statements of identification

Admissions of party opponents- statement introduced against a party that is the party’s own prior statement
1) adoptive admissions
2) vicarious admissions
3) co-conspirators

How well did you know this?
1
Not at all
2
3
4
5
Perfectly
168
Q

hearsay exceptions- declarant unavailable

A
  • former testimony
    -dying declarations
    -statements against interest
    -statements of personal or family history
    -declarant unavailable due to party’s wrongdoing
How well did you know this?
1
Not at all
2
3
4
5
Perfectly
169
Q

hearsay exceptions- declarant unavailability immaterial

A

-present sense impression
-excited utterance
-state of mind (mental emotional or physical condition)
-statements made for purposes of medical diagnosis or treatment
-past recollection recorded
-business records
-public records
-learned treatises
-judgment of previous conviction
-other numerous exceptions
-reputation

How well did you know this?
1
Not at all
2
3
4
5
Perfectly
170
Q

witness qualification

A

preliminary question for the court, not the jury. court not bound by the rules of evidence when determining such questions

How well did you know this?
1
Not at all
2
3
4
5
Perfectly
171
Q

exclude relevant evidence

A

when probative value is sunstnaitally outweighed by a danger of unfair prejudice, confusing the issues, misleading the jury, undue delay, wasting time, or needless cumulation of evidence

How well did you know this?
1
Not at all
2
3
4
5
Perfectly
172
Q

prosecution asking question on hunch

A

question about specific act by defendant allowed, but asking on hunch is bad faith.

How well did you know this?
1
Not at all
2
3
4
5
Perfectly
173
Q

witness impeached with prior conviction involving dishonesty

A

if conviction occurred within the previous 10 years

How well did you know this?
1
Not at all
2
3
4
5
Perfectly
174
Q

child competence to testify

A

depends on child IQ, ability to differentiate between truth and falsehood, and understanding of the important of telling the truth. child must have personal knowledge of a matter to testify about it

How well did you know this?
1
Not at all
2
3
4
5
Perfectly
175
Q

lay witness with personal knowledge

A

may testify as to whether a document is in that person’s handwriting. However, the lay witness may not have become familiar with the handwriting for the purpose of the current litigation

How well did you know this?
1
Not at all
2
3
4
5
Perfectly
176
Q

remedial measure admissibility

A

evidence inadmissible if undertaken AFTER incident. evidence admissible if undertaken BEFORE incident

How well did you know this?
1
Not at all
2
3
4
5
Perfectly
177
Q

rule against hearsay

A

evidence generated by machine or animal does not implicate this rule

How well did you know this?
1
Not at all
2
3
4
5
Perfectly
178
Q

judgments of acquittal

A

NOT a hearsay exception

How well did you know this?
1
Not at all
2
3
4
5
Perfectly
179
Q

congress taxing power- indirect

A

allowed so long as:
1) imposed uniformly in every state where the taxed goods are found AND
2) reasonably related to revenue raising on its face

How well did you know this?
1
Not at all
2
3
4
5
Perfectly
180
Q

peremptory challenges

A

used to strike potential jurors without reason or explanation, unless they are used to exclude based solely on race, ethnicity, or sex

How well did you know this?
1
Not at all
2
3
4
5
Perfectly
181
Q

fifteenth amendment

A

prohibits state and federal Govs from abridging the right to vote- by direct limitation or diluting voting power on the basis of race color, or previous condition of servitude

How well did you know this?
1
Not at all
2
3
4
5
Perfectly
182
Q

shifting executory interest

A

divests an estate held by another grantee such that it shifts from one grantee to another

How well did you know this?
1
Not at all
2
3
4
5
Perfectly
183
Q

springing executory interest

A

divests an estate held by a grantor such that the estate springs from the grantor to the executory interest holder

184
Q

defamation

A

(libel - written) (slander- spoken)
requires proof of the following:
1) D knowingly made a false statement about the P or negligently failed to determine falsity
2) statement was defamatory, the type that would tend the harm the P’s reputation
3) D intentionally or negligently communicated it to third party
4) statement caused P to suffer special damages or amounted to slander per se (where special harm not required bc serious bad statement)

185
Q

receiving stolen property

A

statutory crime when a person:
-receivers control of stolen personal property
-knows that the property was stolen (via larceny embezzlement false pretenses)
-specifically intends to permanently deprive owner of the property

186
Q

anticipatory repudiation

A

may be retracted if nonrepudiation party is notified of the retraction before 1) canceling contract, 2) materially changing position in reliance on the repudiation, or 3) indicating that he/she considers the repudiation to be final

187
Q

Nonjusticiable political question

A

when: 1) the constitution reserves the issue to another branch of government and/or 2) the court lacks judicial discoverable and manageable standards to resolve the issue

188
Q

Congress

A

sole power to impeach and remove federal judges, President, VP, officers (US AG)

189
Q

factual impossibility

A

never a defense to attempted crime. because attempted requires specific intent to commit a crime, performed an act in further, but did not complete it. this is true even when unknown condition makes it factually impossible to complete intended crime.

190
Q

nonconforming use

A

property used in lawful manner before a zoning law was enacted or amended and the use continues even though it is now prohibited.

it may continue until 1) owner enlarges, changes, or abandons that use or 2) if provided in the zoning law, an amortization period has passed or the use has not been registered

191
Q

challenges to jurors

A

three peremptory for any reason except race/ethnicity/ gender

unlimited for cause (bias or impartiality = relationship w party or financial stake in party)

192
Q

ESI

A

sanctions appropriate if party failed to preserve ESI that:
1)should’ve been preserved in anticipation or conduct of litigation
2) is lost bc the party failed to take reasonable steps to preserve it
3) cannot be replaced or restored w additional discovery

if intentionally, court may:
- presume lost ESI was unfavorable to party that failed to save it
-instruct jury that it may or must presume ESI was unfavorable to party or
-dismiss action or enter default judgment against that party

193
Q

rescission of contract

A

available due to mutual mistake if 1) mistake relates to a basic assumption of K, 2) mistake materially affects the agreed upon exchange of performances, and 3) adversely affected party did not assume the risk of the mistake

194
Q

real covenant binding successors if:

A

1) in writing
2) intent to run w land to successors in interest
3) touches and concerns the land- relates to land
4) horizontal privity- promising parties transferred and conveyed simultaneously
5) vertical privity- successor of both
6) notice

title by adverse possession cannot enforce covenant

195
Q

summaries to prove contents of voluminous writing

A

need proper foundation for admitting. must establish:
-originals were voluminous
-originals would be admissible
-originals or copies made available to all parties to examine and
-summary is fair and accurate

196
Q

doctor patient privilege

A

not recognized by fed law, but by state so relevant in diversity situations. inapplicable where:
-patient physical condition in issue
-communication was part of commission of crime or tort
-dispute exists between patient doc
-patient contractually waived

197
Q

fed court standing

A

if allegedly 1) suffered injury in fact 2) caused by D’s challenged conduct and 3) redressable by favorable judicial decision

198
Q

deed

A

if not delivered during grantor’s lifetime then it is void and conveys no interest. grantor right to recover = no delivery bc no present intent to transfer

199
Q

defamation statement to third party privilege

A

privileged when D reasonably believes 1) statement affects important interest of third party and 2) communication socially acceptable. privilege lost if D knows statement is false or recklessly disregards possibility (substantial doubts about truth)

200
Q

attempt

A

occurs when D
-has specific intent to commit crime
-performs act in furtherance of it but
-does not complete it

201
Q

exclusionary rule

A

not applied to grand jury proceedings

202
Q

first amendment

A

guarantees press and public the right to attend every stage of criminal trial. but not absolute and court can order proceedings closed if satisfies strict scrutiny aka show that closure is the least restrictive means to serve compelling government interest

203
Q

statute of frauds

A

MY LEGS
marriage, year (cannot be completed in a year) Land, Executor, Goods, Suretyship. Lease and easements less than a year do not fall under

204
Q

commercial speech

A

that is not false or misleading or concerns unlawful activity receives 1A protection. And any government regulation of that must 1) directly advance a substantial government interest and 2) be narrowly tailored to achieve that interest

205
Q

state constitutions may offer greater protections than federal constitution

A

fed constitution is the floor

206
Q

diversity JX amount in controversy

A

single P may aggregate but costs and interest are excluded from calculation

207
Q

multiparty multiforme trial jurisdiction act MMTJA

A

fed court original JX over action in which minimal diversity if meets requirements

208
Q

common law conspiracy

A

requires proof of at least two guilty minds (plurality of agreement) so the D cannot be convicted of conspiring with persons who feigned agreement

209
Q

juvenile conviction as character evidence

A

never allowed in civil case to attack witness character for truthfulness

210
Q

absent public records exception to hearsay

A

testimony by public official that a diligent search failed to disclose a public record is admissible to prove that the record does not exist, so long as the public office regularly kept records for a matter of that kind

211
Q

best evidence rule

A

does not apply to testimony about failed efforts to find public record, because no record is being introduced into evidence

212
Q

impeached by prior inconsistent statement

A

by intrinsic evidence (elicited from witness sought to be discredited) and extrinsic (from any other source than witness testimony)

213
Q

waive attorney client privilege

A

only client can waive by:
-voluntarily disclosing to third party
-voluntarily giving it up (contractually)
-failing to timely claim privilege
-failing to object to another’s disclosure of confidential communications

not waived by:
death
termination of attorney client relationship

214
Q

habit evidence

A

person’s habit or organization routine practice evidence admissible to prove that person/org acted in accordance with habit/practice on a particular occasion. habit = routine reaction to specific set of circumstances that is semiautomatic in nature and is considered highly probative of the person’s conduct on a particular occasion

215
Q

impeach witness truthfulness

A

an arrest alone does not qualify as a bad act that is admissible to attack a witness’s character for truthfulness

216
Q

expert opinion

A

an expert opinion based upon otherwise inadmissible facts or data is admissible if experts in that particular field would reasonably rely on the facts or data in forming an opinion on the subject

217
Q

answer amended

A

once as a matter of course within 21 days:

after serving answer, if no responsive pleading OR

if responsive pleading required, after being served with responsive pleading or motion, whichever comes first

218
Q

JMOL

A

can be on any claim or defense

219
Q

involuntary dismissal

A

A defendant can move for an involuntary dismissal if the plaintiff failed to prosecute the case or comply with a rule or court order. If the motion is granted, the case will be dismissed with prejudice (unless the court states otherwise) to prevent the plaintiff from suing the defendant on the same claim in the future.

220
Q

types of pleadings

A

complaint, answer, third party complaint, reply

221
Q

reply

A

Federal Rule of Civil Procedure 7(a) lists the pleadings allowed in federal court. The two most common pleadings are the plaintiff’s complaint against the defendant and the defendant’s answer in response to that complaint. A plaintiff need only respond to the defendant’s answer by filing a reply in the rare instance that the court orders the plaintiff to do so.

If the court orders a reply, the plaintiff may be ordered to respond to any allegation in the answer—including an affirmative defense (e.g., novation). Any allegation in the answer that requires a response and is not denied in the reply is deemed admitted by the plaintiff. But when a reply is not required (as seen here), then the plaintiff is deemed to deny any allegation in the defendant’s answer

222
Q

final judgment rule

A

When an action involves multiple claims or parties, a district court may enter final judgment as to fewer than all claims or parties if it expressly determines that there is no just reason for delay. However, if this express determination is not made, any court order that disposes of those claims or parties is not immediately appealable.

223
Q

process

A

Process can be served by (1) following the rules of the state where the court is located or where service is made, (2) delivering the summons and complaint to the defendant personally (or to his/her authorized agent), or (3) leaving the summons and complaint at the defendant’s dwelling with a resident of suitable age and discretion.

224
Q

class certification appeal

A

An appellate court has discretion to permit an appeal from an order granting or denying class action certification if the petition for such an appeal is filed with the clerk of the appellate court within 14 days after the order is entered.

225
Q

final judgment rule exception

A

The final-judgment rule generally precludes federal appellate courts from hearing an appeal until the federal district court has entered a final judgment. But the interlocutory appeals statute (28 U.S.C. § 1292) provides exceptions to this rule that allow certain equitable orders to be immediately appealed as a matter of right. These include:

orders granting, modifying, refusing, or dissolving injunctions

orders appointing or refusing to appoint receivers and

decrees determining the rights and liabilities of the parties to admiralty cases in which appeals from final decrees are allowed.

226
Q

choice of law in diversity cases

A

A federal court sitting in diversity must apply state law to substantive issues and federal law to procedural issues. When it is unclear whether an issue before the court is substantive or procedural, the court must determine if there is a conflict between state and federal law with respect to the issue.

227
Q

permissive joinder

A

Under the permissive joinder rule (FRCP 20), multiple persons may be joined as plaintiffs or defendants in the same suit if:

the claims asserted by or against the joined parties arise out of the same transaction, occurrence, or series of transactions or occurrences and

the action will involve a common question of law or fact among all joined parties.

228
Q

Erie analysis

A

Under this analysis, the issue will be considered substantive and state law will therefore apply if it is outcome determinative AND there is no countervailing federal interest. State law is outcome determinative if the failure to apply state law would result in either:

forum-shopping – litigants will be encouraged to sue in federal court to take advantage of benefits not afforded in state court or

inequitable administration of the laws – the application of substantially different rules in federal and state court would cause unfair outcomes.

229
Q

notice of appeal deadline

A

A party may only appeal, absent limited exceptions, after the district court has entered a final judgment. However, when an action involves multiple claims or parties, a district court may enter final judgment as to fewer than all claims or parties if it expressly determines (i.e., certifies) that there is no just reason for delay. A party may then appeal that judgment if the notice of appeal is filed with the district court clerk within 30 days after entry of that judgment.*

*The time to file the notice of appeal is extended to 60 days if one of the parties is the United States, a federal agency, or a federal officer or employee sued in an official capacity (see image above).

230
Q

diversity cases choice of law

A

When a federal court’s subject-matter jurisdiction is based on diversity (as seen here), the court must apply state law to substantive issues and federal law to procedural issues. When it is unclear whether an issue is substantive or procedural—e.g., penalizing a party for losing an appeal—the court must determine if there is a conflict between state and federal law. If a conflict exists, then the federal-rule analysis provides that a valid federal rule that directly addresses the issue applies if the rule:

is arguably procedural and
does not abridge, modify, or enlarge a substantive right.

231
Q

issue preclusion

A

Issue preclusion (i.e., collateral estoppel) bars the relitigation of issues that were actually litigated, determined, and essential to a valid final judgment—e.g., when the court grants summary judgment (Choice B). There are two types of issue preclusion:

Mutual – when issue preclusion in the second action is asserted by parties to the first action against other parties to the first action

Nonmutual – when issue preclusion in the second action is asserted by nonparties to the first action against parties to the first action

Both types of issue preclusion can be asserted against parties to the first action or their privies, absent limited exceptions (not seen here). However, they cannot be asserted against a nonparty to the first action because it would deprive that party of the opportunity to fully and fairly litigate his/her claims and defenses.

232
Q

excusing juror

A

A juror must participate in the verdict unless excused by the court for good cause during trial or after jury deliberations have begun. Good cause exists when the juror has an illness or family emergency or has committed juror misconduct that might cause a mistrial.

improper for judge to excuse without good cause

233
Q

sanctions

A

A court can impose sanctions, including an order to pay the opposing party’s attorney’s fees, on a law firm (default), attorney, or party for violating one of the provisions of FRCP 11(b)—shown above (Choice D). Sanction proceedings can be initiated in two ways:

by a party’s motion – which requires the party to serve the motion on the alleged violator but refrain from filing it for 21 days after serving it to allow any violation to be corrected (i.e., safe-harbor rule) or

on the court’s own initiative (i.e., sua sponte) – which requires the judge to issue an order to show cause to the alleged violator.

234
Q

impleader

A

Third-party practice (i.e., impleader) allows a defendant to add a nonparty who may be liable to the defendant for all or part of the plaintiff’s claim. However, the court must have original subject-matter jurisdiction or supplemental jurisdiction over the third-party claim.

235
Q

jury verdict

A

A jury verdict must be unanimous and returned by a jury of at least 6 (but no more than 12) members unless the parties stipulate otherwise.

236
Q

relation back doctrine

A

An amended complaint will “relate back” to the date of the original complaint if (1) the same occurrence is at issue, (2) the new party received notice of the suit within 90 days after the original complaint was filed, and (3) the new party knew or should have known that it would have been sued but for a mistake about its identity.

237
Q

appeal

A

If the appellate court permits the appeal, the district court proceedings are stayed pending the appeal only when ordered by the district court or the appellate court.

238
Q

best evidence rule

A

real or physical evidence not subject to it

239
Q

post trial juror testimony

A

Posttrial juror testimony is admissible if it concerns (1) extraneous prejudicial information brought to the jury’s attention, (2) an outside influence improperly brought to bear on a juror, or (3) a mistake made in entering the verdict onto the verdict form.

240
Q

recorded recollection

A

The hearsay exception for recorded recollections allows a record to be read into evidence if it:

concerns a matter that a witness once knew but cannot recall at trial
was made or adopted by the witness when the matter was fresh in his/her mind and
accurately reflects the witness’s personal knowledge at the time it was made.

However, the record may be received as an exhibit only if it is offered by an adverse party (here, the defense). Therefore, the court should not admit the witness’s statement offered by the prosecutor.

241
Q

pleas, plea discussions, and related statements

A

A defendant’s statements during plea negotiations are generally inadmissible against the defendant. However, a defendant may waive this protection if the waiver is made knowingly and voluntarily.

242
Q

witness character for truthfulness

A

A witness’s character for truthfulness can be attacked with a specific instance of conduct (SIC) involving a criminal conviction for a felony or crime of dishonesty—regardless of whether the SIC is introduced intrinsically or extrinsically. However, a SIC involving a mere bad act can only be introduced intrinsically.

243
Q

present refresh recollection

A

Once a witness has used a writing to refresh his/her recollection, the adverse party is entitled to (1) have the writing produced for inspection, (2) cross-examine the witness about the writing, and (3) introduce into evidence any portion of the writing that relates to the witness’s testimony.

244
Q

presentation and evaluation of evidence

A

The court must decide preliminary questions of fact related to whether evidence is admissible, a privilege exists, or a witness is qualified. Any hearing on these matters must be conducted outside the jury’s presence if (1) the matter involves the admissibility of a confession, (2) a defendant in a criminal case is a witness and so requests, or (3) justice so requires.

245
Q

impeaching w criminal conviction

A
246
Q

authenticating photo

A

Tangible evidence must be authenticated before it can be admitted at trial. This requires a prima facie showing that the object is what the proponent claims it to be. A photograph is authenticated by having a witness with personal knowledge—i.e., knowledge based on firsthand observation or experience—of the thing depicted testify that the photograph fairly and accurately depicts that thing.

247
Q

then-existing state of mind hearsay exception

A

Hearsay is an out-of-court statement offered to prove the truth of the matter asserted therein and is generally inadmissible. But statements of the declarant’s then-existing state of mind—e.g., statements of motive, present intent, or plan—are excepted from hearsay. This is true regardless of whether the declarant is available as a witness (Choice A). And since these statements are excepted from hearsay, they can be used as substantive evidence to prove that the declarant later acted in conformity with that state of mind.

248
Q

authenticating- xrays

A

All tangible evidence must be authenticated before it can be admitted at trial. This requires a prima facie showing that the object is what the proponent claims it to be. For example, a simple photograph is authenticated by having a witness with personal knowledge of the thing depicted testify that the photograph fairly and accurately depicts that thing.

But when the evidence sought to be admitted is a physical representation of something that could not otherwise be seen—e.g., an electrocardiogram or x-ray image (as seen here)—authentication requires proof that:

the process for creating the evidence was accurate

the machine that produced the evidence was working properly and

the operator of the machine was qualified to operate it.

249
Q

relevance dependent on existence of fact

A

When the relevance of evidence depends on whether a fact exists, proof must be introduced to support a finding that the fact does exist. The court may admit the proposed evidence on the condition that such proof be introduced later (Choice B). And in determining whether sufficient proof has been introduced, the court must examine all of the evidence and decide whether the jury could reasonably find the conditional fact by a preponderance of the evidence.

250
Q

presentation and evaluation of evidence

A

Although the jury ultimately determines the weight and credibility of evidence, the judge must first determine whether that evidence can be presented to the jury (i.e., admitted) under the Federal Rules of Evidence (FREs) (Choices C & D). Under FRE 801, an out-of-court statement is attributable to a party—and is therefore admissible nonhearsay—if the statement:

was made by the party’s coconspirator
during and in furtherance of the conspiracy.
When determining if these preliminary facts are met, the court is not bound by the FREs (except with respect to privileges). Therefore, the court may consider any relevant evidence tending to establish the facts—even evidence that is otherwise inadmissible.

251
Q

judicial notice

A

Federal Rule of Evidence 201 allows a court to take judicial notice—on its own initiative or upon a party’s request—of any adjudicative fact that is not subject to reasonable dispute because it:

is generally known within the territorial jurisdiction of the trial court or

can be accurately and readily determined from sources whose accuracy cannot reasonably be questioned.

In a civil case, the court must instruct the jury that it is required to accept the noticed fact as conclusive. But in a criminal case, the court should instruct the jury that it may accept the noticed fact as conclusive.

Upon a party’s timely request, the judge must give that party an opportunity to be heard on the propriety of taking judicial notice and the nature of the fact to be noticed. However, a judge is not required to provide this opportunity before taking judicial notice of an adjudicative fact.

252
Q

compromise offers and negotiations

A

Public policy encourages the settlement of disputes. As a result, FRE 408 generally bars the admission of evidence of compromise offers (or acceptance of such offers), as well as conduct or statements made during compromise negotiations, when that evidence is offered to:

prove or disprove the validity or amount of a disputed claim or
impeach by a prior inconsistent statement or contradiction.
This is true even when the party seeking to introduce such evidence was not a party to the agreement.

253
Q

MPC test for insanity

A

at time of conduct, as a result of mental disease/defect, did not have substantial capacity to appreciate the wrongfulness of the act or to conform to the law

254
Q

common law murder

A

unlawful killing of another w malice aforethought

255
Q

causation for murder

A

actual- but for
proximate- foreseeable as result of conduct

256
Q

depraved heart murder

A
257
Q

withdraw from crime

A

CL- never a defense
fed/majority- only after the agreement but before overt act by 1) communicating notice of intent to not participate to other co conspirators or 2) informing police about agreement
MPC/minority- only by acting voluntarily to twart

258
Q

felony murder

A

Felony murder is an unintended killing proximately caused by and that occurs during the commission or attempted commission of an inherently dangerous felony (e.g., robbery). The underlying felony, as the lesser included offense, typically merges into the felony-murder charge.

259
Q

attempt

A

A defendant is guilty of attempt if he/she (1) had the specific intent to commit a crime and (2) took a substantial step toward the commission of that crime. The crime of attempt is complete once the defendant takes the substantial step, so there can be no abandonment or withdrawal after that step is taken.

factual impossibly is never a defense

260
Q

fourth amendment standing

A

A defendant has standing to challenge a Fourth Amendment search if he/she had a legitimate expectation of privacy (or an ownership/possessory interest) in the area or item searched at the time of the search. Therefore, a defendant cannot challenge an unlawful search of a home that was conducted after the defendant moved out.

261
Q

sixth amendment right to counsel

A

The Sixth Amendment right to counsel attaches to offenses that have been formally charged (e.g., by indictment). In Texas v. Cobb, the U.S. Supreme Court held that this right also encompasses offenses that—even if not formally charged—would be considered the same offense as a charged offense. Under the Blockburger test, offenses are not the same if each offense requires proof of an element that the other does not.

The Sixth Amendment right to counsel automatically attaches once formal judicial proceedings have commenced (e.g., at an arraignment). Once the right has attached, it applies to all critical stages of the prosecution.

262
Q

criminal assault

A

Criminal assault occurs when a defendant (1) attempts to commit a battery or (2) intentionally places another in reasonable apprehension of imminent harmful or offensive contact.

263
Q

conspiracy

A

Since the common law follows the bilateral approach to conspiracy, a conviction requires proof of at least two guilty minds. For this reason, and because of the need for consistency in verdicts returned by a single jury,* a conspirator cannot be convicted if all other coconspirators are acquitted at the same trial.*

264
Q

initial aggressor

A

An initial aggressor can claim self-defense only if (1) the aggressor’s use of nondeadly force was met with deadly force or (2) the aggressor, in good faith, completely withdrew from the altercation and communicated that fact to the victim.

265
Q

grand jury

A

A grand jury determines whether probable cause exists to formally charge a person with a crime and consists of citizens who reside in the judicial district where the grand jury sits. Inherent in the concept of a grand jury is that it is representative of the community. As a result, the Fourteenth Amendment equal protection clause prohibits racial discrimination in the selection of grand jurors.

A defendant indicted by a grand jury that excluded members of a racial group may raise an equal protection challenge—even if the defendant is not a member of the excluded group. If the appellate court finds that there was deliberate discrimination in the selection of grand jurors, automatic reversal of the conviction is required

266
Q

Sixth amendment right to counsel

A

Critical stages”

Post-indictment: lineups, in-person identifications, interrogations
Arraignments, preliminary hearings, bail hearings, pretrial motions
Plea negotiations & hearings
Trial & sentencing

Noncritical stages:

Pre-charge lineups
Photo-array identification
Fingerprinting, handwriting & voice exemplars, blood samples
Initial appearances, hearings to determine probable cause to detain defendant
Discretionary appeals
Post-conviction proceedings (eg, parole or probation hearing)

The Sixth Amendment right to counsel:

automatically attaches when formal judicial proceedings have commenced (e.g., indictment, preliminary hearing, arraignment) and

guarantees that criminal defendants will have the assistance of counsel during all critical stages of prosecution—i.e., any event where the absence of counsel may prejudice the defendant’s right to a fair trial.

267
Q

No double jeopardy protections

A

(retrial permitted for same offense)

No attachment:
Jury was not impaneled & sworn or
Judge did not begin to hear evidence

Mistrial:
Requested by defendant or
Based on manifest necessity

Appeal:
Appellate court discovered trial error & remanded case

New facts:
Facts necessary for greater offense did not exist at first trial

Guilty plea to lesser offense:
Greater offense was charged at time of plea to lesser offense

268
Q

Fifth Amendment Double Jeopardy

A

The Fifth Amendment double jeopardy clause bars a second prosecution for the same offense once jeopardy has attached—e.g., when the jury is impaneled and sworn in. However, there is no bar to a second prosecution when a mistrial is declared:

at the defendant’s request or with the defendant’s consent or

due to manifest necessity—i.e., a situation rendering it impossible to continue the trial or reach a fair outcome.

One example of manifest necessity is a hung jury—i.e., a jury that cannot reach a unanimous verdict after deliberation.

269
Q

death penalty- felony murder

A

Felony murder is a killing that occurs during the commission or attempted commission of an inherently dangerous felony (e.g., burglary). An accomplice to the underlying felony may also be convicted of felony murder. But if the accomplice did not kill, attempt to kill, or intend to kill, then the death penalty cannot be imposed on the accomplice unless the accomplice:

significantly participated in the commission of the underlying felony and
acted with reckless indifference to human life.

270
Q

grand jury rights and privileges

A

Rights & privileges that may be invoked at grand jury proceedings

Constitutional rights

Privilege against self-incrimination (Fifth Amendment)
Equal protection (Fifth/Fourteenth Amendment)
Congressmen’s privilege not to be questioned about legislative activities (speech or debate clause)
Privileges

Attorney-client privilege
Spousal privilege
Marital communications privilege
Psychotherapist-patient privilege

An indictment is a written statement by a grand jury that formally charges a person with a crime after a finding of probable cause. In determining whether to issue an indictment, the grand jury may subpoena witnesses to testify—including persons whom the grand jury has reason to believe committed the illegal act(s) being investigated. But since a grand jury only has the investigatory power to indict (not the judicial power to convict), witnesses do not enjoy the full complement of constitutional protections during grand jury proceedings.

For example, a grand jury witness may consult with an attorney outside the grand jury room. But there is no constitutional right to have counsel inside the grand jury room—even when the witness is the target of the grand jury investigation (Choice C). There also is no constitutional right to present witnesses at a grand jury proceeding (Choice D). As a result, the court here should not dismiss the indictment because the witness was not denied any constitutional rights.

271
Q

miranda warnings

A

The Fifth Amendment protects suspects from compelled self-incrimination. To protect this privilege, police must give a suspect Miranda warnings before a custodial interrogation. A custodial interrogation occurs when a person is:

in custody – placed under formal arrest or restrained in his/her freedom of movement to such a degree that a reasonable person would not feel free to terminate the encounter and

subjected to interrogation – questions, words, or actions directed at a suspect that an officer knows or should know are likely to elicit an incriminating response.

If police fail to issue Miranda warnings when they are required, any statements made by the suspect during the custodial interrogation cannot be used against the suspect at trial. However, in Illinois v. Perkins, the U.S. Supreme Court held that Miranda warnings are not required when a suspect does not know that the interrogator is a police officer.

272
Q

M-Naghten

A

mental disease or defect- did not know nature of quality of the act OR the wrongfulness of the act

273
Q

accomplice liability

A

Accomplice liability is imposed when a person provides even slight aid or encouragement to the principal with the specific intent that the encouraged crime be completed—even when that assistance is not ultimately necessary to complete the crime.

274
Q

false pretenses vs forgery

A

False pretenses occurs if a person knowingly misrepresented a past or present material fact with the specific intent to defraud and thereby obtained title to another’s property. In contrast, forgery is the making of a false writing of apparent legal significance with the specific intent to defraud.

275
Q

criminal mental states

A

When a statute does not specify the requisite mental state, the minimum required mental state according to the MPC is recklessness—i.e., the conscious disregard of a substantial and unjustified risk that:

the material element exists or
the material element will result from the defendant’s conduct.

276
Q

test for competency

A

whether D understood nature of proceedings and whether the D knows how to assist lawyer. if lacking then incompetent at the time of trial

277
Q

in court ID testimony

A

Due process requires the suppression of unreliable testimony identifying the defendant as the perpetrator of the charged crime since it may inhibit the defendant’s right to a fair trial. In-court identification testimony may be unreliable if it stems from unnecessarily suggestive out-of-court identification procedures arranged by police. Therefore, such in-court identification testimony is admissible only if the prosecution demonstrates that it is sufficiently reliable—i.e., poses no substantial likelihood of misidentification.

278
Q

school searches

A

School officials need only have a reasonable suspicion that a student is violating (or has violated) the law or school rules to search that student. Then, the search methods must be (1) reasonably related to the objective of the search and (2) not excessively intrusive considering the student’s age, sex, and the nature of the infraction.

279
Q

automobile exception to search

A

The Fourth Amendment prohibits unreasonable searches and seizures. A search or seizure is unreasonable unless conducted pursuant to a warrant based on probable cause (not seen here) or an exception to the warrant requirement. The automobile exception allows officers to conduct a warrantless search of a vehicle if:

there is probable cause to believe that the vehicle contains evidence of a crime and
the search is limited to areas where the evidence might be located.
During an automobile search, police can search any containers inside the vehicle—including locked containers—that might contain the evidence. But probable cause to believe that a vehicle contains evidence of a crime does not justify a warrantless search of a person who is, or had been, a passenger in the vehicle. Police must have probable cause to believe that evidence is on the passenger before searching the passenger.

280
Q

5A double jeopardy

A

The Fifth Amendment double jeopardy clause generally prohibits a second prosecution after a prior conviction for the same offense. Offenses are considered the same for double jeopardy purposes if one is a lesser included offense of the other. That is because every element of the lesser offense is included in the greater offense. Therefore, when a conviction of a lesser included offense stems from a guilty plea, the double jeopardy clause bars a subsequent prosecution for the greater offense unless:

an event necessary to establish the greater offense occurred after the plea was entered or
the greater offense was charged before the plea was entered.

281
Q

due process- reasonable doubt

A

Due process requires that the government prove every element of a criminal offense beyond a reasonable doubt. Therefore, when the defendant asserts a defense (e.g., alibi) that negates an element of the crime, the burden of proof must remain on the prosecution to prove that element. In other words, the burden may not be shifted to the defendant to disprove the element (Choices A & C). To be proper, a jury instruction must reflect this constitutional requirement.

282
Q

grand jury rights

A

Rights & privileges that may be invoked at grand jury proceedings

Constitutional rights

Privilege against self-incrimination (Fifth Amendment)
Equal protection (Fifth/Fourteenth Amendment)
Congressmen’s privilege not to be questioned about legislative activities (speech or debate clause)
Privileges

Attorney-client privilege
Spousal privilege
Marital communications privilege
Psychotherapist-patient privilege
A grand jury’s function is to determine whether probable cause exists to issue an indictment against an individual. Grand jury proceedings are investigatory—not adversarial. As a result, a target of a grand jury investigation has no constitutional right to confront witnesses or present evidence (Choices C & D). Therefore, the court should not dismiss the defendant’s indictment on the grounds that he was denied the opportunity to confront the neighbor, present a witness, and testify before the grand jury.

283
Q

6A right to impartial jury

A

The Sixth Amendment guarantees a criminal defendant the right to be tried by an impartial jury—i.e., a jury pool selected from a fair cross section of the community and an unbiased impaneled jury. Therefore, a potential juror whose views would substantially impair his/her ability to impartially decide the case can be challenged for cause.

284
Q

confession- unlawful arrest

A

Under the exclusionary rule, an unlawful arrest generally requires suppression of evidence obtained as a result of that arrest. However, an incriminating statement made after an unlawful arrest is admissible if the connection between the arrest and the statement is so attenuated that the statement is considered voluntary. To make this determination, the court will examine the totality of the circumstances, including:

the length of time between the arrest and the statement
the flagrancy of the police misconduct and
the existence of intervening events between the arrest and the statement.
Miranda warnings are not, by themselves, a sufficient intervening event to break the connection between an illegal arrest and a subsequent statement. And a statement that is too closely connected to an unlawful arrest should be suppressed.

285
Q

checkpoint stop

A

A checkpoint to gather information about a specific crime is reasonable if (1) the primary purpose is to apprehend individuals other than the motorists, (2) the stop significantly advances a public concern, and (3) the checkpoint is tailored to fit important criminal investigatory needs and minimally interfere with Fourth Amendment rights.

286
Q

handwriting sample

A

The taking of handwriting exemplars does not violate the Fourth Amendment (no reasonable expectation of privacy), the Fifth Amendment (no protection for physical evidence), or the Sixth Amendment (not a critical stage of prosecution).

287
Q

structural error

A

the erroneous denial of a defendant’s choice of counsel (as seen here) constitutes structural error—i.e., an error that affects the entire framework of a criminal trial, rendering it fundamentally unfair. Structural errors are not subject to review for harmlessness and require automatic reversal of the defendant’s conviction

288
Q

ex post facto clauses

A

Ex post facto clauses

(applies to federal (art. 1, § 9) & state (art. 1, § 10) governments)

Prohibit enactment of retroactive criminal laws that:

criminalize previously legal conduct
impose greater punishment than previously prescribed
eliminate previously available defenses
decrease prosecution’s previous burden of proof
The ex post facto clauses prohibit federal and state governments from enacting criminal laws that have a retroactive punitive effect (i.e., ex post facto laws). As a result, this clause generally does not apply to civil laws because their purpose is nonpunitive. However, a civil law will be deemed to be an ex post facto law when its retroactive effect is so punitive that it clearly overrides its nonpunitive purpose.

Courts weigh several factors to determine whether the punitive effect of a retroactive civil law clearly overrides its nonpunitive purpose. These include whether the law:

imposes an affirmative disability or restraint (e.g., imprisonment) (not seen here)
has historically been regarded as punishment (e.g., public shaming)
promotes the traditional aims of punishment (e.g., retribution)
is rationally related to its nonpunitive purpose* most important factor

289
Q

discriminatory laws

A

Discriminatory laws can be challenged under the equal protection clause and are generally subject to rational basis review, which requires that a law be rationally related to a legitimate government objective. But heightened scrutiny is used when a law substantially impacts a fundamental right or intentionally discriminates against a suspect or quasi-suspect class. A law can intentionally discriminate:

on its face – the language of the law distinguishes between different classes (intent presumed)
in its application – a facially neutral law is purposefully applied differently to different classes or
in its motive – an otherwise neutral law was enacted to disproportionately impact a protected class.

290
Q

equal protection- citizenship

A

Under Article I of the Constitution, Congress has plenary (i.e., exclusive) authority over immigration and naturalization, so federal laws based on U.S. citizenship are presumptively constitutional. To rebut this presumption and show that such a law violates the equal protection component of the Fifth Amendment, a challenger must show that the law is not rationally related to a legitimate government interest—i.e., that the law is arbitrary or unreasonable.

291
Q

judge immunity

A

Judges possess absolute immunity from civil liability for official judicial actions—including rulings that are grave procedural errors—unless the court clearly lacked subject-matter jurisdiction at the time the action was taken. This ensures that judges are free to issue decisions during judicial proceedings without fear that they will later be personally liable for them.

292
Q

first amendment

A

The First Amendment freedom of the press generally prohibits the government from restricting the right to publish lawfully obtained, truthful information about matters of public significance. A government action that abridges this right is presumptively unconstitutional and must survive strict scrutiny. This requires the government to prove that its action was the least restrictive means (i.e., narrowly tailored) to achieve a compelling government interest.

Here, the trial court issued a gag order that prohibited the press from reporting about court proceedings, which abridged the freedom of the press. The court likely had a compelling interest in preventing the release of sensitive information that might impair ongoing undercover investigations, but it had less restrictive means available to achieve this interest—e.g., postponing the trial. Therefore, the appellate court should overturn the gag order because it cannot survive strict scrutiny and is likely unconstitutional (Choices B & D).

Additionally, the First Amendment generally guarantees the press and the public the right to attend every stage of a criminal trial. However, this right is not absolute, and a court can order that trial proceedings be closed if the court’s findings on the record demonstrate that the closure withstands strict scrutiny.

Here, the court prohibited the public from attending the trial to protect the identities of witnesses, who feared violent repercussions if they testified. Although this promoted the compelling interest of protecting witness safety, less restrictive means were available to serve this interest—e.g., allowing witnesses to testify outside the presence of the public. Therefore, the appellate court should overturn the order for a closed trial because it cannot survive strict scrutiny and is likely unconstitutional

293
Q

equal protection

A

Government actions that treat similarly situated persons differently (i.e., discriminate) can be challenged under the Fourteenth Amendment equal protection clause. These challenges are typically subject to rational basis review. This standard of review presumes that the government’s actions were constitutional. As a result, the challenger must show that the law has no rational relation to any legitimate government interest—i.e., that the law is arbitrary or invidiously discriminatory.

294
Q

enumerated congressional powers

A

Congressional legislation must stem from Congress’s enumerated powers. Under the militia clauses, Congress has the power to authorize the President to call state National Guard units into action without the approval of the state governor to execute federal laws, suppress insurrections, or repel invasions. This power extends to the use of state National Guard units in domestic situations and non-emergency circumstances.

295
Q

1A right to associate

A

The First Amendment protects against government interference with a person’s right to associate with any group or organization. But since this right is not absolute, the government can punish (i.e., deny public employment to or criminally prosecute) persons who:

are active members of a subversive organization
know of the organization’s illegal objectives and
specifically intend to further those objectives.

296
Q

free exercise clause

A

The First Amendment free exercise clause prohibits government interference with a religious organization’s right to shape its faith and mission. As a result, the ministerial exception protects religious organizations from civil liability for employment discrimination when they hire or fire employees who serve in ministerial roles.

297
Q

fundamental rights - First VIP

A

first amendment freedoms, voting, interstate travel, Privacy (marriage/family, parental rights, sexual acts, contraceptives)

Under the Fourteenth Amendment equal protection clause, a discriminatory law that substantially impacts the fundamental right to vote is only justified if it survives strict scrutiny. This occurs in the rare instance that the state proves that the law is necessary to achieve a compelling state interest. However, the U.S. Supreme Court has held that, under Section 2 of the Fourteenth Amendment, states may prohibit felons—even those unconditionally released from prison—from voting in elections. Therefore, the court will likely uphold the statute as constitutional.

298
Q

eleventh amendment- state immunity

A

Immunity:
Suit brought by private party or foreign government
Suit against state official violating state law

Exceptions:
State consents to suit
Immunity removed by 13th, 14th, or 15th Amendment
State official sued for injunctive or declaratory relief for alleged constitutional violation
Damages to be paid by state officer personally (not state treasury)
State official sued for prospective (not retroactive) damages to be paid by state treasury

No immunity:
Suit brought by United States or other state
Suit against local government (eg, counties, municipalities)
Bankruptcy proceedings

299
Q

Younger Abstention doctrine

A

The Younger abstention doctrine requires that a federal court abstain from issuing a declaratory judgment or injunction if doing so would interfere with a pending state criminal, or particular civil, proceeding that (1) involves an important state interest and (2) provides an adequate opportunity to litigate federal issues.

300
Q

enumerated congressional powers

A

Taxing & spending
Interstate & foreign commerce
War, armed forces, militia
Coin & borrow money
Immigration & naturalization
Mail
Copyright & patent laws
Federal courts
District of Columbia
Bankruptcy
Rules concerning captures

301
Q

wealth discrimination

A

State laws that discriminate against individuals based on wealth (e.g., ability v. inability to pay a court fee) can be challenged under the Fourteenth Amendment equal protection clause. Since an individual’s wealth is not a suspect class or quasi-suspect class, such laws are generally subject to rational basis scrutiny and upheld. However, courts will depart from this test and apply strict scrutiny when a state law prohibits the exercise of a fundamental right (e.g., the right to appeal) based on an individual’s wealth.

302
Q

14A neutral decision maker

A

The Fourteenth Amendment due process clause requires that the government provide an opportunity to be heard before a neutral decision-maker. A judge must therefore recuse him/herself from a case when (1) the judge has a direct, personal, substantial, pecuniary interest in it or (2) a serious, objective risk of actual bias exists.

303
Q

scotus review of state court decisions

A

The Supreme Court of the United States (SCOTUS) may use its discretionary appellate jurisdiction and grant a writ of certiorari to review final state-court judgments. But since SCOTUS may only review such judgments if they are based on federal law, it cannot review a judgment based on adequate and independent state grounds. This occurs when:

state law fully resolves the matter, so the outcome of the case is not affected by federal law (adequate) and

the state court did not rely on federal law to reach its decision (independent).

304
Q

redistricting

A

Article I of the Constitution grants states the power to draw state and federal legislative districts. However, when exercising this power, states must comply with other constitutional provisions. This includes the Fourteenth Amendment equal protection clause, which requires that race not be the predominant factor used to draw boundary lines for state or federal legislative districts. If it is, then the boundary lines will be subject to strict scrutiny review and likely deemed unconstitutional.

305
Q

export clause

A

The export clause prohibits federal taxation of:

exported goods, which are goods leaving the U.S. and shipped to foreign countries and
services and activities closely related to the export process.

306
Q

enclave clause

A

power over DC

307
Q

civil forfeiture

A

In civil forfeiture actions, the government deprives an individual of his/her property interest by seizing property allegedly used in criminal activity. As a result, procedural due process generally requires that the government provide reasonable notice of the seizure and a meaningful opportunity to be heard before a neutral decision-maker. However, personal property may be seized prior to providing notice and a hearing when:

the seizure serves a significant government interest
that interest would be frustrated by advance notice of the seizure and
the seizure is performed by the government.

308
Q

5A takings

A

A Fifth Amendment taking occurs when the government destroys private property or property rights—including possessory and nonpossessory interests in land (e.g., easements, liens).

And under the Fifth Amendment, a taking must be for a public use (e.g., fighting forest fires), and the government must justly compensate the owner for the property’s fair market value.

309
Q

1A freedoms

A

First Amendment freedoms

Expression-Right to communicate information & ideas through speech or conduct

Association-Right to form or participate in any group or organization

Press-Right to publish truthful information

Religion:
-Free exercise – right to practice religion or no religion without government interference

Establishment – prohibits government from favoring particular religion or religion generally

The First Amendment shields the media from criminal and civil liability for publishing lawfully obtained private facts (e.g., crime victim’s identity) and other truthful information involving matters of public concern (i.e., newsworthy events). This amendment also shields the media from liability for publishing truthful information that was unlawfully obtained by a third party if:

the information involves a matter of public concern and

the publisher neither obtained it unlawfully nor knows who did.

310
Q

congress delegation

A

Congress can delegate incidental legislative powers to federal agencies if it provides an intelligible principle to guide the agency—i.e., a clear statement defining (1) the policy Congress seeks to advance, (2) the agency to carry out that policy, and (3) the scope of that agency’s authority.

311
Q

conditional federal funding

A

(Article I, section 8 spending clause)

Conditions must meet all the following criteria:

clearly stated & unambiguous
reasonably related to federal interest in funded program
do not require states to engage in unconstitutional activity
do not unduly coerce states into accepting

The spending clause gives Congress broad power to spend for the general welfare (i.e., for any public purpose) (Choice B). Congress may use its spending power to incentivize state or local governments to act in certain ways by placing conditions on their receipt of federal funds. But those conditions are binding only if they are clearly stated and unambiguous so that potential recipients may make an informed decision.

312
Q

fed immunity for state taxation

A

Under the supremacy clause, the federal government is immune (i.e., exempt) from any direct taxation by the states—including taxes on its commercial activities. As a result, states may not impose taxes on the federal government, or any of its agencies or instrumentalities, without the express consent of Congress.

313
Q

twenty first amendment

A

The Twenty-first Amendment grants states broad authority to regulate alcohol within their borders. This authority includes the ability to prohibit the importation, transportation, or sale of alcohol within the state and to delegate such authority to local governments (e.g., municipalities).

314
Q

substantive due process

A

Substantive due process principles are based upon the idea that laws should be reasonable and not arbitrary.* As a result, states cannot deprive individuals of life, liberty, or property without adequate justification. The required level of justification depends on whether the law deprives the individual of:

an ordinary right – under the rational basis test, the law is presumed valid until the challenger shows that the law has no rational relation to any legitimate government interest or

a fundamental right – under the strict scrutiny test, the law is presumed invalid until the government shows that the law is necessary to achieve a compelling government interest.

315
Q

elections clause

A

The Article I, section 4 elections clause grants state legislatures the power to enact laws that regulate the time, place, and manner of congressional elections (e.g., by establishing voting sites). But the clause also grants Congress the power to override those state laws by supplanting them with federal law.

316
Q

state electoral elections

A

State-election restrictions

Restriction

Examples

Test

Ordinary

(nondiscriminatory)

Voter registration
Photo-ID requirement
Disallowing write-in voting
Rational basis: challenger must show restriction lacks rational relationship to legitimate state interest

Severe

(discriminatory)

Poll tax
Disallowing third-party candidacies
Property-ownership requirement
Strict scrutiny: state must show restriction is necessary to achieve compelling state interest

States may regulate their own elections by requiring voters and candidates to abide by certain requirements. But these regulations must comply with the First Amendment and the Fourteenth Amendment equal protection clause, which together secure an individual’s right to vote and freely associate—e.g., the right to access the ballot. If an electoral regulation is challenged under either constitutional provision, the standard for evaluating that regulation depends on the severity of the burden imposed. A court will apply either:

rational basis review – applies to reasonable, nondiscriminatory burdens and requires the challenger to prove that the regulation is not rationally related to a legitimate state interest or

strict scrutiny – applies to severe or discriminatory burdens and requires the state to prove that its regulation is the least restrictive means of achieving a compelling state interest.

317
Q

appointment of federal officers

A

(Art. II, § 2 appointments clause)

Type

Examples

Appointment

Principal

Ambassador
Supreme Court justice
Cabinet official
Appointed by President (with Senate approval)

Inferior

Independent counsel
Judicial clerk
Administrative law judge
Appointed by President (with Senate approval) unless Congress delegates appointment to:

President alone
federal courts or
heads of executive departments
The Article II appointments clause grants the President the power to appoint principal federal officers with the Senate’s advice and consent (i.e., approval). But Senate approval is not required for the President’s appointment of inferior federal officers—i.e., federal officers who are supervised by principal federal officers. That is because Congress may delegate the appointment of inferior federal officers to the President alone (i.e., without Senate approval), the heads of executive-branch agencies, or the federal courts.

318
Q

14A due process parents’ rights to control

A

The Fourteenth Amendment due process clause generally prohibits states from enacting laws that substantially impair parents’ right to control their children’s upbringing and education. However, this right is not absolute, so states may impose reasonable educational standards on schools without violating the due process clause.

319
Q

commerce power to regulate- substantially affect IC

A

The commerce clause grants Congress broad power to regulate interstate commerce—including activities (even in-state activities) that substantially affect interstate commerce. When determining if activities have a substantial effect on interstate commerce that warrants regulation by Congress, courts consider whether:

the activities are economic in nature (if so, a substantial effect is presumed)

the regulation has a jurisdictional element that limits its reach to activities with a direct connection to or effect on interstate commerce

there are express congressional findings concerning the activities’ effect on interstate commerce and

there is a strong link between the regulated activities and that effect.

320
Q

federal legislative action

A

Once Congress delegates power to an executive agency, it cannot interfere with the agency’s functions without satisfying the legislative-action process—i.e., bicameralism and presentment. Any attempt to bypass this process is an unconstitutional legislative veto.

321
Q

dormant commerce clause

A

The Article I commerce clause empowers Congress to regulate interstate commerce. It also carries a negative implication (i.e., the dormant commerce clause), which prohibits states from (1) discriminating against out-of-state commerce or (2) otherwise unduly burdening interstate commerce. A discriminatory state law—i.e., one that favors in-state over out-of-state interests—will be deemed unconstitutional unless the state proves that:

the law furthers an important or legitimate noneconomic state interest (e.g., health, safety) and
there is no reasonable, nondiscriminatory alternative to achieve that interest.

The dormant commerce clause generally prohibits states from discriminating against or unduly burdening interstate commerce. But states can do so when Congress has explicitly authorized that conduct.

322
Q

congress power citizenship

A

Congress has plenary (i.e., exclusive) power to regulate naturalization—i.e., the process through which noncitizens obtain U.S. citizenship. But the Fourteenth Amendment limits this power by prohibiting Congress from revoking the citizenship of any U.S. citizen without his/her consent unless that citizenship was obtained by fraud or in bad faith.

323
Q

appointment of federal officers

A

A federal officer is someone who (1) holds a continuing public office and (2) has significant discretionary authority to administer or enforce laws (i.e., executive powers). All federal officers must be appointed by the President or in a manner otherwise consistent with the Article II appointments clause. Under this clause, Congress cannot appoint federal officers because Congress’s participation in this executive function would violate the separation of powers doctrine.

324
Q

second amendment

A

The Second Amendment—applicable to states and municipalities through the Fourteenth Amendment due process clause*—generally prohibits government interference with an individual’s right to possess and use firearms for a traditionally lawful purpose (e.g., self-defense in one’s home). However, this right is not absolute (Choice A). SCOTUS has stated that certain government restrictions on firearms are permissible, including:

banning unusually dangerous firearms (e.g., grenades)
imposing conditions and qualifications on commercial sales of firearms
forbidding the possession of firearms by felons and mentally ill individuals and
prohibiting the open carry of firearms in certain areas (e.g., schools) or concealed carry of firearms in public.

325
Q

article 4 property clause

A

The Article IV, section 3 property clause gives Congress complete power to dispose of and regulate federally owned land and territories. This clause includes the power to regulate private property that affects federal public lands when such regulation is necessary to protect those lands—e.g., by prohibiting the erection of structures that completely enclose federally owned land. Therefore, the court is likely to rule for the federal government.

326
Q

self authenticating docs- notarized

A

proponent of notarized need not give advance notice to adverse party of intent to introduce doc

327
Q

best evidence rule`

A

Overview of best evidence rule

General rule

Must produce original or reliable duplicate* to prove contents of document relied on by witness or whose contents are at issue

Exceptions

Original unavailable

Other evidence can be used to prove content if:

originals lost or destroyed (not by proponent’s bad faith)
originals not attainable by judicial process
opponent had original, knew it was required & failed to produce or
content not closely related to controlling issue
Admission by party

Contents can be proven by opposing party’s testimony, deposition, or written statement

Public record

Contents of public record can be proven by:

certified copy
copy of records & comparison testimony or
other evidence if above proof is not reasonably obtainable
*Duplicates are admissible unless original’s authenticity is questioned or it would be unfair to admit them.

328
Q

civil judgment hearsay exception

A

NO SPECIFIC hearsay exception for civil judgments

329
Q

witness prior inconsistent statement- for impeachment

A

A prior inconsistent statement is a past statement that is contrary to a witness’s present testimony. Such statements may generally be used to impeach a witness by:

examining the witness about the statement or

introducing the statement through extrinsic evidence if (1) the witness has an opportunity to explain or deny—and the opposing party has the opportunity to question the witness about—the statement or (2) justice so requires.

However, a party generally may not impeach the credibility of a witness by introducing extrinsic evidence of a collateral matter—i.e., a matter that is irrelevant to the outcome of the case. Instead, the party must accept the witness’s testimony.

330
Q

victim sex behavior in civil case

A

Evidence of a victim’s other sexual behavior or sexual predisposition is generally inadmissible in a civil proceeding involving sexual misconduct. But such evidence may be admitted if the court determines that the probative value of the evidence substantially outweighs the danger of harm to the victim and unfair prejudice to any party.

331
Q

handwritten document authentication

A

The authenticity of a handwritten document may be disputed by allowing (1) a lay witness with personal knowledge of the authentic handwriting (acquired outside of the current litigation) to give opinion testimony on the disputed handwriting or (2) an expert witness or the trier of fact to compare the authenticated and disputed handwriting.

332
Q

curative admission

A

Under the doctrine of curative admission, when inadmissible evidence is improperly admitted against a party, the court may permit that party to introduce additional inadmissible evidence for the purpose of rebuttal. This is meant to remedy the prejudicial effect caused by the previously admitted evidence.

333
Q

excited utterance / present sense impression

A

exceptions to hearsay:

An excited utterance is a statement (1) relating to a startling event or condition and (2) made while the declarant was under the stress of excitement caused by that event or condition. A present sense impression is a statement (1) describing an event or condition and (2) made while or immediately after the declarant perceived it.

334
Q

statements in treatises- hearsay exception

A

Under the rule against hearsay, an out-of-court statement is inadmissible when offered to prove the truth of the matter asserted therein unless a hearsay exclusion or exception applies. One exception applies to statements in a learned treatise, periodical, or pamphlet when:

the statements are called to the attention of or relied on by an expert witness during examination and

the publication is established as a reasonably reliable authority by a party’s expert or judicial notice (Choice B).

Such statements can be used as substantive evidence to help establish a material fact. However, a party may only have the particular statements read into evidence—the treatise itself cannot be received as an exhibit.

335
Q

public records- hearsay exception

A

Under the rule against hearsay, an out-of-court statement offered to prove the truth of the matter asserted therein is inadmissible absent an exception or exclusion. One exception applies to existing public records, while another applies to the absence of public records. Under the latter exception, testimony by a public official that a diligent search failed to disclose a public record or statement is admissible to prove that:

the record or statement does not exist or
a matter did not occur, if a public office regularly kept a record for a matter of that kind.
In a criminal case, the prosecution must provide the defense with written notice of its intent to offer such evidence at least 14 days before trial. The defendant then has seven days from receipt of notice to object in writing.

336
Q

best evidence rule

A

The best evidence rule requires that an original or reliable duplicate of a recording, writing, or photograph (referred to as “document”) be produced to prove its content. However, a party may use other, secondary evidence (e.g., testimony) to prove a document’s content if certain factual conditions are established (e.g., all originals are lost or destroyed).

Whether a party has fulfilled the factual conditions for admitting secondary evidence is a determination for the court. But in a jury trial, the jury determines any issue about whether:

an asserted document ever existed
another document produced at a hearing or trial is the original or
other evidence of content accurately reflects the content.

337
Q

cross examination of criminal defendant

A

Under Federal Rule of Evidence 104, preliminary questions relating to the admissibility of evidence must be decided by the court. This includes questions regarding the admissibility of a confession in a criminal case. To decide these questions, the court may hold a hearing and receive testimony (as seen here).

When a criminal defendant testifies at such a hearing, the defendant is subject to cross-examination about the specific issue addressed at the hearing—here, the voluntariness of the defendant’s confession—and issues related to the defendant’s credibility. This allows the court to fully assess the issues and properly rule on the admissibility of the evidence.

However, the criminal defendant is not subject to cross-examination on other issues in the case. This allows the defendant to participate in the determination of preliminary questions—which often involve the defendant’s constitutional rights—without being subjected to cross-examination generally.

338
Q

criminal defendant good character evidence

A

Under FRE 404, a criminal defendant may offer evidence of his/her good character when that trait is pertinent to the charged crime. For example, evidence of a defendant’s peacefulness is admissible when the defendant is charged with a violent crime. But truthfulness is not pertinent to violent crimes like murder, so evidence of this trait should not be admitted on this basis (Choice C).

However, evidence of a criminal defendant’s truthfulness also may be admissible if the defendant testifies at trial—even when that trait is not pertinent to the charged crime. FRE 608 allows the admission of reputation or opinion testimony to prove the witness’s truthful character after it has been attacked. But here, there is no indication that the defendant’s character for truthfulness had been attacked, so testimony regarding that character remains inadmissible.

339
Q

federal inadvertent waiver rule

A

The federal inadvertent-waiver rule applies to (1) communications covered by the attorney-client privilege and (2) materials protected under the attorney work-product doctrine. Under this rule, disclosure of a communication protected by the attorney-client privilege or material that qualifies as attorney work product does not operate as a waiver in a federal or state proceeding if:

the disclosure was made in a federal proceeding or to a federal agency
the disclosure was inadvertent
the privilege holder had taken reasonable steps to prevent disclosure and
the privilege holder promptly took reasonable steps to rectify the error.
Materials qualify as attorney work product if they were prepared by or for an attorney in anticipation of litigation—as seen with the investigator’s report.

340
Q

evidence of liability insurance

A

Under Federal Rule of Evidence 411, evidence of liability insurance (or the lack thereof) is inadmissible to prove negligence or wrongdoing. This public-policy rule helps ensure that the jury will not infer fault based on the existence or lack of coverage and will not base its verdict on the defendant’s or insurer’s ability to pay. However, such evidence is admissible for other purposes, such as:

impeaching a witness with evidence of self-interest, bias, or prejudice or
proving agency, ownership, or control of the insured item.

341
Q

treatise statements

A

Statements in a learned treatise, periodical, or pamphlet are excepted from the rule against hearsay when (1) the publication is established as a reliable authority by a party’s expert or judicial notice and (2) the statements are called to the attention of or relied on by an expert witness during examination.

342
Q

NOT HEARSAY

A

An out-of-court statement is not hearsay if offered for a purpose other than to prove the truth of the matter asserted—e.g., to show that a party had notice of a relevant fact or condition.

343
Q

reply letter doctrine

A

A document can be authenticated by establishing that it qualifies as a reply letter. To do so, the proponent must show that (1) the document was written in response to an earlier communication and (2) the contents make it unlikely that it was written by someone other than the recipient of the earlier communication.

344
Q

1A right to free speech

A

The First Amendment right to free speech, applied to the states through the Fourteenth Amendment, does not extend to public employees speaking pursuant to their official duties. A public employee speaks pursuant to his/her official duties when that speech is made within the scope of the public employee’s ordinary job duties (e.g., drafting an internal memorandum).

But when a public employee’s speech is made outside the scope of his/her ordinary job duties, the public employee is speaking as a private citizen—even if that speech concerns those job duties. When a public employee is speaking as a private citizen on a matter of public concern, the government can limit that speech only if its interest in efficient government function outweighs the employee’s right to free speech.

When a public employee is speaking pursuant to official duties, the government has great latitude to regulate that speech. But when the employee is speaking as a private citizen, the government can restrict that speech only if its interest in efficient government function outweighs the employee’s right to free speech.

345
Q

serving process in US

A

A defendant must receive notice of a lawsuit through service of process—i.e., delivery of the summons and complaint. Under FRCP 4, a plaintiff can properly serve any defendant (individual or organization) when someone who is at least 18 years old and not a party to the suit serves the defendant with a summons and complaint. When the defendant is an organization (e.g., a corporation), FRCP 4(h) permits service in the United States by either:

following the service-of-process rules of the state where the court is located (i.e., forum state) or where service is made or
delivering process to an officer, managing or general agent, or agent authorized by appointment or law to receive service AND, if required by statute, mailing process to the defendant.

346
Q

time to be served

A

within 90 days of plaintiff filing complaint

347
Q

counsel at probation-revocation when

A

Due process requires counsel at a probation-revocation proceeding if the person (1) denies committing the alleged violation or (2) asserts that complex reasons justified or mitigated it. The Sixth Amendment guarantees this right if a sentence for the underlying offense (1) has not been imposed and (2) will be imposed if the probation is revoked.

348
Q

collateral order doctrine

A

Examples of collateral orders

District court orders:
denying state 11th Amendment immunity
denying government official/employee immunity
vacating attachment of vessel
refusing to require security bond pursuant to state law
remanding action to state court based on abstention

The collateral-order doctrine allows an immediate appeal from an interlocutory order that (1) conclusively resolves an important issue that (2) is separate from the merits of the claim and (3) cannot be effectively reviewed on appeal from a final judgment—e.g., an order denying a claim of governmental immunity.

349
Q

article IV property clause

A

The Article IV property clause grants Congress complete power to dispose of and regulate federally owned property. As part of this property power, Congress may enact statutes regulating the use of buildings owned by the federal government—e.g., by converting uninhabited, federally owned buildings into post offices

350
Q

answers to interrogatories

A

Answers to interrogatories are nonhearsay party admissions. And when such an answer incorporates a document—e.g., by referring the interrogating party to the document—the contents of the document are considered adopted by the answering party. This means that the contents are nonhearsay if offered against the answering party. However, admission of the document itself must comply with other evidentiary rules.

351
Q

conspiracy

A

A conspiracy conviction requires proof that the defendant had the specific intent to accomplish an unlawful purpose—even if the intended crime does not require specific intent (e.g., when the intended crime is a strict liability offense).

352
Q

lack of diversity citizenship

A

Diverse citizenship does not exist when a suit involves stateless persons—i.e., (1) noncitizens present in the U.S. but not citizens of a foreign country or (2) U.S. citizens domiciled in a foreign country.

353
Q

SCOTUS jurisdiction

A

Original jurisdiction is a court’s power to hear and decide a case first—i.e., before any appellate review. Article III provides that the U.S. Supreme Court (SCOTUS) has original jurisdiction over (1) cases involving foreign ambassadors, public ministers, or consuls and (2) cases in which a state is a party.

Congress may not expand or restrict SCOTUS’s original jurisdiction, but Congress may determine the cases within SCOTUS’s original jurisdiction that are exclusive to it. As a result, Congress has granted SCOTUS exclusive jurisdiction over controversies between two or more states, which occur when states seek redress for injuries directly caused by other states. However, Congress has denied SCOTUS exclusive jurisdiction over all other cases within its original jurisdiction by granting lower federal courts concurrent jurisdiction over:

cases involving foreign ambassadors, public ministers, or consuls
cases between the United States and a state and
cases between a state and citizens of another state or noncitizens.

354
Q

inconsistent verdicts

A

Inconsistent verdicts are those that are logically incompatible. For example, a jury that finds a defendant not guilty of one offense but guilty of another that was predicated upon commission of the first offense has rendered an inconsistent verdict. In such circumstances, the guilty verdict cannot be challenged based upon the inconsistency (Choice C). The rationale behind this rule is that the jury may have reached its decision to acquit through mistake, compromise, or leniency and that its decision should not be disturbed.*

*While a challenge based solely on verdict inconsistency is not permitted, a criminal defendant may still challenge the conviction on the ground that the evidence was legally insufficient to convict.

355
Q

redline doctrine

A

Under the Redline doctrine (followed in most jurisdictions), a defendant is not guilty of felony murder when a cofelon dies at the hands of a victim or the police. However, this doctrine does not bar conviction for depraved-heart murder.

356
Q

first amendment media shield

A

The First Amendment shields the media from criminal and civil liability for publishing lawfully obtained private facts (e.g., crime victim’s identity) and other truthful information involving matters of public concern (i.e., newsworthy events). This amendment also shields the media from liability for publishing truthful information that was unlawfully obtained by a third party if:

the information involves a matter of public concern and
the publisher neither obtained it unlawfully nor knows who did.

357
Q

voluntary dismissal of claims

A

Absent the parties’ stipulation, a party may voluntarily dismiss a crossclaim, counterclaim, or third-party claim without a court order by unilaterally filing a notice of dismissal (1) before a responsive pleading is served or (2) if a responsive pleading is not served, before evidence is introduced at a hearing or trial.

358
Q

14A voting in states

A

The Fourteenth Amendment due process and equal protection clauses protect the fundamental right to vote—including voting in local elections (e.g., city council elections) (Choice D). State or local laws that substantially impact the right to vote can be challenged under either clause and are usually subject to the strict scrutiny test. This difficult test requires the state to show that its law is the least restrictive means (i.e., necessary) to achieve a compelling state interest (Choice B).

However, courts will depart from this test and apply rational basis scrutiny when a state or local law limits the right to vote in a governmental unit’s election to its residents—i.e., those who reside within the governmental unit’s borders. Since this test presumes that the law is constitutional, the challenger must prove that the law has no rational relation to any legitimate state interest.

359
Q

false pretenses

A

A defendant is guilty of false pretenses if he/she:

obtained title to (i.e., ownership of) another’s property (e.g., cash)
by knowingly misrepresenting a past or present material fact upon which the victim relied and
did so with the specific intent to defraud.
To prove reliance, the prosecution must establish that the defendant’s misrepresentation was a significant factor in—or the cause of—the victim’s decision to pass title to the property to the defendant.

360
Q

search warrant

A

A search warrant is presumptively valid if it (1) is issued by a neutral and detached magistrate based on probable cause, (2) is supported by a sworn oath or affidavit, and (3) particularly describes the place to be searched and items to be seized. However, a defendant may invalidate the warrant if the defendant can prove by a preponderance of the evidence that:

the application contained a false statement necessary to the finding of probable cause and
the officer submitting the application knew the statement was false or recklessly disregarded its falsity.

361
Q

enumerated congress powers

A

The necessary and proper clause allows Congress to take actions that are reasonably necessary to carry out its enumerated Article I powers. Article I, Section 8 enumerates the extremely broad war and defense powers that permit Congress to declare war and to provide for the national defense during wartime and peacetime—e.g.:

by excluding civilians from sensitive or military areas
by implementing a military draft and
by imposing wage, price, and rent controls on the private civilian economy (as seen here).
Therefore, the federal rent-control statute enacted during wartime is likely constitutional pursuant to the congressional war and defense powers.

362
Q

5A compelled to testify

A

A person subpoenaed to testify before a grand jury who has invoked the Fifth Amendment privilege against self-incrimination can be compelled to testify if he/she is granted use and derivative use immunity. This prevents the person’s testimony, and any evidence derived from it, from being used against him/her in any way that could lead to a criminal prosecution. But the government may still prosecute the person using evidence obtained from independent sources (as seen here).

However, in New Jersey v. Portash, the U.S. Supreme Court held that testimony given under a grant of immunity is coerced and therefore involuntary. And any criminal-trial use of such testimony outside the context of a perjury prosecution constitutes a denial of due process. As a result, it is not constitutionally proper for the prosecutor to impeach the mayor’s testimony with a statement that he made before the grand jury.

363
Q

impeaching with criminal conviction

A

A conviction for a felony not involving dishonesty that is not more than 10 years old is admissible against a criminal defendant-witness if the conviction’s probative value outweighs its prejudicial effect.

364
Q

specific jurisdiction

A

Long-arm statutes specify when a court within the forum state can exercise specific personal jurisdiction over a nonresident defendant (e.g., manufacturer, holding company). Here, State A’s long-arm statute authorizes jurisdiction over out-of-state defendants to the fullest extent that is constitutionally permissible (Choice A). However, due process only permits application of a long-arm statute when:

the plaintiff’s claim arises from or is closely related to the defendant’s minimum contacts with the forum state and
the exercise of jurisdiction complies with notions of fair play and substantial justice.
Minimum contacts exist when the defendant purposefully avails itself of the state’s protections and benefits so that it should reasonably foresee being sued there—e.g., by advertising and selling goods in the state. The minimum contacts of a subsidiary may be imputed to its parent corporation when the subsidiary is acting as the parent corporation’s agent or alter ego. But in the absence of such evidence, each corporation is a separate legal entity.

365
Q

Wharton rule

A

A criminal offense that requires two parties cannot be the object of a conspiracy that consists of two parties.

366
Q

battery = general intent crime

A

general intent crime that includes not only intentional conduct but also criminal negligence

367
Q

eighth amendment excessive bail clause

A

Eighth Amendment Excessive Bail Clause requires that, when bail is set, it cannot be excessive, i.e., set higher than an amount reasonably calculated to ensure the defendant’s presence at trial. Since the amount of the bail in this instance is reasonably calculated to ensure the defendant’s presence at trial, it does not violate this clause

368
Q

government burden of proof- affirmative defense

A

A state (or the federal government) may place the burden of proving an affirmative defense, such as self-defense, on the defendant without violating the Due Process Clause. The preponderance of the evidence standard for judging whether the defendant has met this burden is also constitutional. The jury instructions, however, improperly prevent the jury from considering the defendant’s self-defense evidence, not as an affirmative defense, but as a defense to the elements of the crime that the prosecution must prove. Used in such a manner, this evidence need not satisfy the preponderance standard before being considered by the jury.

369
Q

exclusionary rule

A

The primary remedy for Fourth Amendment violations is the “exclusionary rule,” which prevents the introduction at a subsequent criminal trial of evidence unlawfully seized. The remedy provided by the exclusionary rule generally applies only to criminal trials; it is not applicable to civil proceedings such as wrongful death actions.

370
Q

altering jury awards

A

A party may move for a new trial on the basis that the fact finder awarded an inadequate amount of damages. If a federal court rules that the amount was inadequate, it may order a new trial but may not impose an increase in the amount of damages (i.e., additur).

371
Q

default judgment

A

A court clerk must enter a default judgment when (1) the plaintiff’s claim is for a sum certain or a sum that can be made certain by calculation, (2) the plaintiff’s request for default judgment includes an affidavit establishing the amount due, (3) the defendant failed to appear, and (4) the defendant is not legally incompetent or a minor.

372
Q

right to jury trial

A

The Seventh Amendment right to a jury trial applies to federal civil cases when the amount in controversy exceeds 20 dollars and a party asserts a legal claim (i.e., a claim seeking a monetary remedy). However, this right does not apply when a party asserts an equitable claim (i.e., a claim seeking a nonmonetary remedy).

373
Q

two dismissal rule

A

Under the two-dismissal rule, a voluntary dismissal is with prejudice when the plaintiff (1) voluntarily dismissed an action in federal or state court without a court order and (2) filed a notice of voluntary dismissal in a second action on the same claim in federal court.

374
Q

claim aggregation amount-in-controversy

A

Federal Rule of Civil Procedure (FRCP) 18 allows a party to join (i.e., add) as many claims as it has against an opposing party in a single lawsuit. But like all disputes in federal court, a suit involving multiple joined claims must satisfy subject-matter jurisdiction. This can be established through either:

federal-question jurisdiction – when a claim arises under the U.S. Constitution, a federal treaty, or a federal law (not seen here) or

diversity jurisdiction – when the opposing parties are citizens of different states and the amount in controversy exceeds $75,000.

If subject-matter jurisdiction is based on diversity jurisdiction, then a party may aggregate (i.e., combine) all claims to be joined against a single opposing party to satisfy the amount-in-controversy requirement—even if those claims are unrelated

375
Q

venue for suit against fed officer or official

A

Official capacity

(recovery from government)

Special venue rules permit suit in district where:

any Δ resides
substantial portion of events occurred or property located
π resides if no real property involved
Individual capacity

(recovery from officer/employee)

General venue statute permits suit in district where:

any Δ resides, if all Δs reside in same state
substantial portion of events occurred or property located
any Δ subject to personal jurisdiction if neither of above applies

The general venue statute applies when a federal officer or employee is sued in his/her individual capacity. But special venue rules apply when a federal officer or employee is sued for acting in an official capacity or under color of legal authority.

376
Q

choice of law

A

When a federal court’s subject-matter jurisdiction is based on diversity jurisdiction (as seen here), the court must apply federal law to procedural issues and state law to substantive issues. Certain issues are clearly substantive, including the statute of limitations, the applicable burden of proof, as well as the existence and elements of a claim or defense. Therefore, the district court must recognize the affirmative defense in the State B statute—not the federal act

377
Q

summary judgment

A

A court should grant a motion for summary judgment when:

there is no genuine dispute of material fact and

the evidence is legally insufficient for a reasonable jury to find in the nonmovant’s favor, so the movant is entitled to judgment as a matter of law.

In ruling on this motion, the court must (1) construe all evidence in the light most favorable to the nonmovant and (2) resolve all doubts in favor of the nonmovant.

378
Q

sanctions safe harbor rule

A

A violation of one of the provisions of Federal Rule of Civil Procedure (FRCP) 11(b) (shown above) can subject a law firm, attorney, or party to sanctions. Sanction proceedings can be initiated by the court sua sponte or by a party’s motion. When sanction proceedings are initiated by a party’s motion, the safe-harbor rule requires the party serving the motion to refrain from filing it for 21 days after serving it to allow any violation to be corrected (e.g., by dismissing a claim).

379
Q

general verdict with special interrogatories

A

In a general verdict with special interrogatories, the jury decides which party should prevail and provides answers to questions on each factual issue. If the verdict and answers are inconsistent, the judge must (1) order a new trial, (2) direct the jury to further consider its answers and verdict, or (3) enter a judgment consistent with the answers.

380
Q

impleader rules

A

Impleader allows a defendant to add a nonparty (i.e., third-party defendant) to a suit who may be liable to the defendant for all (i.e., indemnity) or part (i.e., contribution) of the plaintiff’s claim. When this occurs, the plaintiff may assert his/her own claim against the third-party defendant only if that claim:

arises out of the same transaction or occurrence that is the subject matter of the plaintiff’s claim against the defendant in the original complaint and

satisfies original subject-matter jurisdiction on its own (see image above).

Here, the defendant properly impleaded the third party for indemnity (Choice C). The spouse then sought leave to amend her complaint to add a claim against the third party for negligence. Although the spouse’s claim against the third party arises out of the decedent’s death, the spouse and the third party are citizens of the same state (Choice D). Therefore, the court lacks diversity (and subject-matter) jurisdiction over the claim.

However, when a claim falls outside a court’s original jurisdiction, the court may exercise supplemental jurisdiction if the supplemental claim and the original claim share a common nucleus of operative facts (as seen here). But when the original claim is based on diversity jurisdiction, supplemental jurisdiction is barred if the supplemental claim:

is made by (1) existing plaintiffs against parties added through joinder, intervention, or impleader or (2) persons seeking to join as plaintiffs through compulsory joinder or intervention and

would violate the requirements of diversity jurisdiction (e.g., contaminate diversity of citizenship).

381
Q

forum non conveniens

A

Forum non conveniens is a common-law doctrine that allows a court to dismiss an action—even if personal jurisdiction and venue are otherwise proper—if the court finds that the forum would be too inconvenient for parties and witnesses, and that another more convenient venue is available. The appropriate way to enforce a forum-selection clause pointing to a state or foreign forum is to seek dismissal through the doctrine of forum non conveniens. Therefore, this is the most likely remedy available to the court in this case.

382
Q

venue- case removed from state court

A

In a case that is removed from state court, venue is proper in the federal district court in the district where the state action was pending. Since this action was removed from a state court in the Northern District of State A, venue is proper in the federal district court for the Northern District of State A.

383
Q

minimum contacts test - corporations & independent contractors

A

In order for a defendant to be subject to the jurisdiction of a court, the defendant must have sufficient minimum contacts with the forum state. While a single contact can be sufficient if the cause of action is based on that contact, an out-of-state corporation is not subject to personal jurisdiction solely because of contacts in the state by an independent contractor. Contacts by a nonresident employer’s agents or employees, on the other hand, are generally imputed to the employer when the agent or employee is acting within the scope of the agency or employment. In this case, the truck driver was acting within the scope of his employment by transporting the equipment through State A when the accident occurred. However, if the corporation can establish that the truck driver was an independent contractor rather than an employee, then the court will not have general jurisdiction over the corporation due to the corporation’s lack of business dealings in State A and would not have specific jurisdiction over the corporation with respect to the accident.

384
Q

sanctions for failure to disclose emails

A

Under Rule 37, if a party fails to make the automatic disclosures required by Rule 26(a) or fails to respond to discovery that has been properly served, the party seeking the information may move to compel such disclosure or discovery. Generally, making a motion to compel is a prerequisite to obtaining any sanctions under Rule 37. In this case, the bicycle manufacturer failed to disclose the emails that it planned to use to support its defense. This information was subject to automatic disclosure under Rule 26(a). However, the bicyclist did not first serve a motion to compel on the bicycle manufacturer prior to seeking sanctions—a prerequisite to obtaining sanctions under Rule 37. Therefore, the court should not grant the bicyclist’s request.

Initial disclosures under Rule 26(a)(1), such as the emails here, must be made at or within 14 days after the parties’ discovery conference, unless a different time is set by stipulation or court order. In this case, the facts indicate that the bicycle manufacturer still had not disclosed the emails 21 days after the discovery conference.

385
Q

issue preclusion - parties

A

One of the requirements for the use of issue preclusion is that the party against whom it is to be asserted in the current lawsuit was a party (or one in privity with a party) to the prior lawsuit. In this case, because the passenger was not a party or in privity with a party to the prior lawsuit between the driver-owner and the truck driver, the truck driver may not successfully assert that the determination of his fault with regard to the accident in that lawsuit precludes the litigation of that issue in the current lawsuit.

386
Q

removal statute - venue

A

Under Section 1441(a) of the removal statute, an action may be removed to the federal court in the district where the state action is pending. That removal statute determines venue, regardless of whether venue would have been proper under the venue statute (Section 1391) if plaintiff had originally brought the action in that federal district court.

387
Q

joining defendants in single action

A

In order to join defendants in a single action, the plaintiff’s right to relief must be asserted against them jointly, severally, or in the alternative with respect to or arising out of the same transaction or occurrence.

388
Q

remand order

A

A remand order is generally not reviewable on appeal or otherwise (except for an order remanding a civil rights case removed pursuant to § 1443 or a remand order in a class action, if the application for review is made to the court of appeals not more than 10 days after the entry of the order). Thus, appeal of the remand order would be improper.

389
Q

appellate review

A

A trial judge’s decision to grant a summary judgment motion is a legal ruling. As a legal ruling, the appropriate standard of review by the appellate court is de novo. While an appellate court relies on the record created in trial court and does not entertain the admission of additional evidence, the appellate court reviews the evidence and law without deference to the trial court’s legal rulings.The abuse of discretion standard for appellate court review of a trial judge’s ruling is appropriate when the ruling is a matter over which the trial judge has discretion, such as whether evidence is admissible or sanctions should be imposed. A trial court’s findings of fact, including a master’s findings that have been adopted by the court, are subject to review under the “clearly erroneous” standard. A trial judge’s grant of a summary judgment motion constitutes a legal ruling, so the “clearly erroneous” standard does not apply. is incorrect. When a trial judge gives an erroneous jury instruction to which the adversely affected party failed to make a timely objection, the appellate court may nevertheless consider the matter if the mistake constitutes a plain error that affects the party’s substantial rights; however, this standard does not apply to a trial judge’s grant of a summary judgment motion.

390
Q

leave of court for depositions

A

Under Rule 30, a party may take the deposition of any party or nonparty witness at any time after the party has made its mandatory initial disclosures pursuant to Rule 26(a). Without leave of the court, the plaintiffs and the defendants, each as a group, are limited to 10 depositions by oral or written examination. Unless the parties agree to the deposition, leave of the court must be obtained to (i) exceed the 10-deposition limitation, (ii) depose a witness a second time, or (iii) depose a person before the deposing party has complied with its initial disclosure requirements under Rule 26(a).

391
Q

state law but raises fed question

A

When state law creates a cause of action, a federal court can nonetheless exercise general federal-question jurisdiction if the complaint raises a real and substantial issue of federal law, and the outcome necessarily depends on resolving this federal issue. Here the plaintiff’s complaint raises a genuine and substantial federal law issue (the constitutionality of the federal commercial paper), which must be resolved to decide her request for an injunction.

392
Q

motions to strike pleadings

A

A court, upon motion or upon its own initiative, may order a pleading containing an insufficient defense to be stricken. Here, since the cause of action arose three years ago when the goods were delivered, the seller’s affirmative statute of limitations defense is insufficient as a matter of law.

393
Q

impleader

A

Under Rule 14(a)(1), a defendant alleging that a third person is liable to him for all or part of the plaintiff’s claim against him may implead such a person as a third-party defendant. Such joinder is permissive, not required.

394
Q

summary judgment

A

If a motion for summary judgment is properly made and supported, then an opposing party may not rely merely on allegations or denials in her own pleading, but she must either set out specific facts showing a genuine dispute for trial or show by affidavit or declaration that, for specified reasons, she cannot present facts essential to oppose the motion. If the opposing party does not so respond, then summary judgment, if appropriate, will be entered against that party. In this case, the shopper properly made a motion for summary judgment and supported it with three affidavits, fully supporting the shopper’s version of the event. As such, the shopper has met the burden of persuasion. The grocery store, however, did not submit any affidavits or other evidence demonstrating a genuine issue of fact for trial. The grocery store also failed to assert that, due to an insufficient opportunity to obtain discovery, she could not present evidence that would demonstrate a genuine issue of fact. Rather, the store relied merely on the general denial in the answer. Because the shopper met the burden of persuasion and the grocery store has not presented any evidence to the contrary, the court should grant the shopper’s motion for summary judgment.

395
Q

class action amount in controversy

A

while most class actions require that at least one member of the class have damages in excess of $75,000, there is no requirement under the CAFA that any member of the class have such damages.

In order to take advantage of the relaxed subject matter jurisdictional rules of the Class Action Fairness Act (CAFA), the amount-in-controversy must exceed $5 million

396
Q

delegation of legislative power from Congress to Executive

A

Because Congress is vested by Article I with “all legislative powers,” it may not delegate that power to any other branch of government. This principle is known as the “nondelegation doctrine.” However, delegation of some of Congress’s authority to the executive branch has consistently been held constitutional, so long as Congress specifies an “intelligible principle” to guide the delegate. Certain powers are nondelegable (e.g., power of impeachment, power to declare war). Furthermore, an executive agency may not make “decisions of vast economic and political significance” unless Congress clearly gives it decision-making authority in that area (i.e., the major questions doctrine). Here, the congressional act provided an intelligible standard with which to determine the percentage of vehicular emissions to be enforced by executive order. And though the executive order may have vast economic significance, Congress clearly gave the executive branch decision-making authority in this area. Therefore, the provision at issue it is likely constitutional.

397
Q

congress necessary and proper clause to investigate

A

Congress does not have an express power to investigate, but the Necessary and Proper Clause allows Congress broad authority to conduct investigations incident to its power to legislate. The investigatory power may extend to any matter within a “legitimate legislative sphere.” In this case, the Senate committee’s investigation concerned the failure to prosecute under a federal statute covering unlawful restraint of trade. The enforcement, or lack thereof, of a federal statute covering trade and commerce certainly constitutes a matter within a legitimate legislative sphere of Congress. Therefore, the Senate committee had the power to investigate this issue incident to its power to legislate

398
Q

due process determination

A

When an individual’s protected interest is threatened by governmental action, a court considers three factors in determining the amount of process that is due: (i) the private interest affected by the governmental action; (ii) the risk of erroneous deprivation of that interest using current procedures and the probable value of additional or substitute safeguards; and (iii) the burden involved in providing the additional process. A state has the burden to demonstrate a compelling interest justifying its actions when a fundamental right is at issue.

399
Q

contracts clause

A

A state may impair the obligations of a contract, including a contract entered into by the state with a private citizen, so long as the impairment is not substantial, or, if the impairment is substantial, so long as it is not unreasonable. In the case of a contract to which the state is a party, the state must show that its important interest cannot be served by a less restrictive alternative and that the impairment it seeks is necessary because of unforeseeable circumstances. Here, the impairment is arguably not substantial because the contractor is receiving the full contract amount plus interest to compensate him for the delay in receiving the money. Assuming that the impairment is substantial, the state’s actions are reasonable in light of the unforeseen budget crisis and arguably there is not a less restrictive alternative

400
Q

SCOTUS writ

A

dismiss if state decision rests on adequate and independent state grounds

401
Q

eleventh amendment

A

The Court has interpreted the Eleventh Amendment as barring unconsented private suits against a state for retroactive money damages. Congress may abrogate state immunity from liability if it is clearly acting to enforce rights created by the remedial provisions of the Thirteenth, Fourteenth, and Fifteenth Amendments (i.e., the Civil War Amendments), and does so expressly. Here, Congress is not remedying conduct that violates the Fourteenth Amendment’s Equal Protection Clause, as age is not a suspect or quasi-suspect class and thus a state’s age-based discrimination merely needs a rational basis.

402
Q

public employment due process

A

here is a legitimate property interest in continued public employment only if there is an employment contract or a clear understanding that the employee may be fired only for cause. An “at will” governmental employee has no right to continued employment. With regard to the remaining week in his one-year term, the employee was paid in full and therefore did not suffer an economic harm as a consequence of his early termination. With regard to future employment, the employee had no assurances from his employer that he would be given permanent employment. Therefore, denying the employee an opportunity to be heard with regard to his termination does not violate his due process rights

403
Q

equal protection based on one’s status as a lawful resident noncitizen

A

Classifications based on status as a lawful resident of the United States (as opposed to a citizen) are subject to a variety of different standards, depending on the level of government and the nature of the classification. Courts will generally apply the strict scrutiny test and strike down state laws that discriminate against noncitizens, such as laws prohibiting noncitizens from owning land, obtaining commercial fishing licenses, or being eligible for welfare benefits or civil service jobs. A growing exception exists, however, for state laws that restrict or prohibit a noncitizen’s participation in government functions. Such laws need only have a rational relationship to a legitimate state interest, and are generally upheld as preventing noncitizens from having a direct effect on the functioning of the government. Because working as a law enforcement officer is a position involving participation in the governmental function of enforcing law, rational basis is likely the proper standard to apply

404
Q

import export clause of article I

A

he Import-Export Clause of Article I, Section 10 prohibits the states from imposing any tax on any imported or exported goods, or on any commercial activity connected with imported goods. The Commerce Clause limits states from taxing commerce with other countries because it vests in Congress the exclusive power to regulate foreign commerce. Therefore, the statute would conflict with Congress’ exclusive right to regulate foreign commerce.

405
Q

speech or debate clause

A

The Speech or Debate Clause of Article I, Section 6 protects members of Congress from civil and criminal liability for statements and conduct made in the regular course of the legislative process, including a speech given on the floor of Congress, committee hearings, and reports. However, this protection does not foreclose prosecution for a crime, including the taking of bribes, when the crime does not require proof of legislative acts or inquiring into the motive behind those acts. Here, the member solicited and accepted a bribe regarding the lowering of a federal excise tax on tobacco products. This crime does not require proof of a legislative act or the motive behind the member’s report, so he is subject to criminal liability

406
Q

billboard content based regulation question

A

Compelled removal of the billboards’ message constitutes a content-based regulation of speech. The state may only regulate the content of speech if the regulation is necessary to achieve a compelling government interest and is narrowly tailored to meet that interest. Accordingly, there would have to be a compelling governmental interest in taking down the message; this is a test that is incredibly difficult to meet. Content-based regulation is generally only found constitutional when the speech to be regulated falls into one of the following categories: obscenity, subversive speech, fighting words, defamation, or commercial speech. Here, the message does not fall into any of these categories. Answer choice A is incorrect because to be regulated as “fighting words,” the speech must be words that by their very nature are likely to incite an immediate breach of the peace. Taking “any and all measures” is insufficient to support a finding of fighting words. Answer choice B is incorrect because the content of the billboards is not subversive just because it is advocating for the repeal of a law. Subversive speech is speech that is directed to incite or produce imminent lawless action and that is likely to produce such action. Here, the billboards are directing citizens on how to repeal a law through the procedures set forth by the state. The billboards are not seeking to incite lawless action; they are informing those who see the billboards on how to take action under the state’s laws. Answer choice C is incorrect because the media has no greater First Amendment protection than other citizens.

407
Q

state action for conlaw

A

State action may exist if there are sufficient mutual contacts between the conduct of a private party and the government to find that the private party is subject to constitutional restrictions. The symbiotic relationship here between the Board and the association (e.g., overlapping membership, Board oversight, and inclusion of association employees in the state’s health and retirement programs) indicates the association’s actions are pervasively entwined with the Board’s authority and actions. Consequently, this constitutes a strong argument in favor of treating the association as a state actor

while an entity that performs a traditional and exclusive governmental function is a state actor for the purposes of free speech rights, providing athletic programs is not a traditional and exclusive governmental function. Merely providing a product or service that the government could offer is not sufficient to make the provider a state actor.

408
Q

commercial speech restrictions- intermediate scrutiny

A

Restrictions on commercial speech, such as advertising, are subject to intermediate scrutiny and are reviewed under a four-part test: (i) it must concern lawful activity and be neither false nor misleading; (ii) the asserted government interest must be substantial; (iii) the regulation must directly advance the asserted interest; and (iv) the regulation must be narrowly tailored to serve that interest. Although the state’s concern for the health of minors constitutes a compelling interest, the law is not narrowly tailored to serve that interest.

Commercial speech (i.e., advertising and similarly economically oriented expression) is entitled to an intermediate level of First Amendment protection. Restrictions on commercial speech are reviewed under a four-part test. First, the commercial speech must concern lawful activity and be neither false nor misleading. Second, the asserted governmental interest must be substantial. Third, the regulation must directly advance the asserted interest. Fourth and finally, the regulation must be narrowly tailored to serve that interest; this means there must be a “reasonable fit” between the government’s ends and the means chosen to accomplish those ends. Here, if an advertisement that a dentist is a specialist in a non-ADA recognized area of practice is inherently misleading, the speech is unprotected commercial speech

409
Q

intrastate activity regulation

A

With respect to an intrastate activity that does not have a direct economic impact on interstate commerce, as long as there is a rational basis for concluding that the “total incidence” of the activity in the aggregate substantially affects interstate commerce, Congress may regulate even a minute amount of that total. Because the fact pattern provides that there is such a basis here for regulation of individual landowners who cut trees found on their own property to burn for their own personal energy needs, the proposed legislation is likely to be found constitutional as a valid exercise of the congressional power to regulate interstate commerce

410
Q

law hierarchy

A

When a federal statute conflicts with an executive agreement, the federal statute takes precedence over the executive agreement. Here, the federal statute and the executive agreement directly conflict with one another, so the federal statute governs whether the territory can buy bananas from the island nation. even though the President does have the power to enter into executive agreements with foreign nations, the executive agreement is subject to federal statutes. because a federal statute and a federal treaty are roughly equivalent. When the two conflict, whichever was enacted more recently would govern. However, here, the two do not conflict. The federal treaty does not govern whether the territory can buy bananas from the island nation; it only requires that a mark of origin be included on those bananas. federal actions, whether statutes, treaties, or executive agreements, take precedence over state laws.

411
Q

presentment clause

A

The Presentment Clause requires any bill that has been passed by Congress to be presented to the President. If the President signs the bill, it becomes law. If the President does not, it may either become law or not depending on whether the President vetoes it (either directly or via a “pocket veto”), and if by a direct veto, whether Congress overrides the President’s veto. The federal law that effectively gives the President a so-called “line-item veto” thwarts the procedure set out in the Presentment Clause.

412
Q

federal classification based on citizenship

A

Because Congress has plenary power over noncitizens under Article I, a federal classification based on citizenship status is likely valid unless it is arbitrary and unreasonable. Since the reason for the law (i.e., to reduce the deficit) is rational, it is constitutional.

413
Q

juvenile adjudication

A

Evidence of a juvenile adjudication is not admissible to impeach a defendant. When the witness is not the defendant, evidence of a juvenile adjudication can be used to impeach the witness’s character for truthfulness only if:

i) It is offered in a criminal case;

ii) An adult’s conviction for that offense would be admissible to attack the adult’s credibility; and

iii) Admitting the evidence is necessary to fairly determine guilt or innocence.

Under the Sixth Amendment Confrontation Clause, evidence of a witness’s juvenile adjudication can also be used by a criminal defendant to impeach a witness’s credibility by showing bias, such as when the witness’s juvenile adjudication could provide a motive for the witness to lie.

414
Q

tangible evidence authenticated

A

All tangible evidence must be authenticated. To authenticate an item, the proponent must produce sufficient evidence to support a finding that the thing is what its proponent claims it is.

415
Q

preliminary questions for trial judge

A

The trial judge generally decides preliminary questions regarding the competency of evidence, including the admissibility of evidence, whether a privilege exists, and whether a person is qualified to be a witness. Hearings on preliminary matters must be conducted outside the presence of the jury when the hearing involves the admissibility of confessions, when a defendant in a criminal case is a witness and so requests, or when justice requires it. In this case, the defendant sought a hearing outside of the presence of the jury regarding the admissibility of his confession. Accordingly, the court must grant the defendant’s request and the judge will decide whether the confession is admissible. Answer choice A is incorrect. Hearings on preliminary matters must be conducted outside the presence of the jury only when the hearing involves the admissibility of confessions, when a defendant in a criminal case is a witness and so requests, or when justice requires it.

416
Q

character evidence for substantive evidence

A

Character evidence is admissible as substantive evidence when character is an essential element of a claim or defense, rather than a means of proving a person’s conduct. Here, the truth of the student’s allegations is an affirmative defense to the claim of libel, and the student may offer specific instances of the professor’s conduct to support that affirmative defense

417
Q

then-exiting state of mind

A

must relate to present intent, motive, or plan

418
Q

evidence relevancy

A

Evidence is relevant if (i) it has any tendency to make a fact more or less probable than it would be without the evidence (i.e., probative) and (ii) the fact is of consequence in determining the action (i.e., material)

under FRE 403, relevant evidence may be excluded if its probative value is substantially outweighed by the danger of unfair prejudice.

419
Q

supremacy clause

A

state courts cannot challenge the legality of a person being held in federal courts through the issuance of a writ of habeas corpus

420
Q

forum defendant rule

A

prohibits removal solely on the basis of diversity jurisdiction. does not prohinit removal if federal question JX

421
Q

warrant validity

A

A defendant may go beyond the face of a warrant and challenge its validity due to the inclusion of false information in the affidavit. However, the defendant must establish by a preponderance of the evidence that the false information was knowingly, intentionally, or with reckless disregard for its truthfulness included by the affiant in the affidavit and that the information was necessary to a finding of probable cause by the magistrate

422
Q

direct vs indirect evidence (circumstantial)

A

Evidence may be direct or circumstantial. Direct evidence is identical to the factual proposition that it is offered to prove, while circumstantial evidence is evidence that tends to indirectly prove a factual proposition through inference and collateral facts. There is no rule requiring the presentation of direct evidence to convict a criminal defendant. Here, the prosecution has offered only indirect evidence of the defendant’s guilt but this is not a valid basis for a motion to dismiss the charges because a criminal defendant can be convicted solely based on circumstantial evidence. More significantly, whether the testimony is characterized as direct or circumstantial, a defendant may be convicted even though the prosecution only presents circumstantial evidence.

423
Q

parole evidence rule

A

The parol evidence rule operates to exclude evidence that, if introduced, would change the terms of a written agreement. However, only evidence of prior or contemporaneous negotiations is subject to the parol evidence rule; evidence of negotiations conducted after the execution of the written contract is not prohibited by the parol evidence rule and may be offered to prove subsequent modifications. Because the buyer sent the email after the contract was executed, it would not be prohibited by the parol evidence rule.

424
Q

The UCC provides that a merchant seller generally retains the risk of loss in the absence of a contract term to the contrary until the buyer receives the goods. However, if the buyer is in breach of the contract, the risk of loss passes to the buyer to the extent of any deficiency in the seller’s insurance coverage. Here, the store, as buyer, was in breach of the contract by failing to pick up the ornaments by 2:00 pm. Although the UCC only requires that the delivery time be “reasonable” in the absence of a specific contract term, the parties here modified the contract in that regard by agreeing that the seller should pick up the ornaments by 2:00 pm

A
425
Q

Unlike the Fourteenth Amendment and the Contracts Clause, the Thirteenth Amendment does not require state action. Rather, it abolishes slavery and its “badges and incidents,” including racial discrimination in private transactions like contracts. Moreover, Section 2 of the Thirteenth Amendment authorizes Congress to enact legislation to implement its guarantees

A
426
Q

Under the “Dual Sovereignty” doctrine, prosecution of a defendant by the federal government for a crime arising out of an event does not prevent a state from prosecuting the defendant for a crime arising out of the same event. (Note: Under this doctrine, the reverse is also true.) Under Blockburger, robbery and conspiracy to commit robbery are separate offenses.

A
427
Q

The Elections Clause of Art. I explicitly empowers Congress to override state laws concerning federal elections. This express provision makes irrelevant general principles of federalism embodied in the “commandeering” cases.

A
428
Q

The holder of a future interest, such as a remainder interest, has a license to inspect the property for waste. This license is not subject to revocation by the holder of the current possessory interest in the property.

A
429
Q

For a larceny, the initial taking and asportation of another’s property must be trespassory; that is, the defendant must not be legally entrusted with the property.

A
430
Q

In a comparative negligence jurisdiction, assumption of the risk is not recognized as a separate defense—it has been merged into the comparative-fault analysis and merely reduces recovery. The plaintiff’s awareness of the risk of her conduct is generally taken into account in determining the degree to which she is at fault, but it can also be considered in determining the reasonableness of the plaintiff’s or the defendant’s actions

A
431
Q

In general, an offer can be revoked by the offeror at any time prior to acceptance, even if it states it will remain open for a specific amount of time, unless the offer is a firm offer or is made irrevocable by an option contract. If the offeree acquires reliable information that the offeror has taken definite action inconsistent with the offer, the offer is automatically revoked by a constructive revocation. Here, the offer remained revocable because it was not a firm offer under the UCC or an option supported by valid consideration.

A
432
Q

A witness may be compelled to provide incriminating testimony if the government grants him immunity from prosecution. “Use” immunity prohibits only the use of the compelled testimony against the witness. If the government does prosecute the witness in such a case, the government has the burden to show that the compelled testimony did not provide an investigatory lead that was helpful to the prosecution

A
433
Q

A court may not grant a motion for a judgment as a matter of law when the motion is made for the first time after the jury has rendered its verdict.

A
434
Q

A tenant is free to sublet the leasehold unless the landlord and tenant agree otherwise. Any lease clause that purports to limit this right is narrowly construed

A
435
Q

Under the Restatement (Second) of Contracts, a third party must be a vested intended beneficiary of a contract in order to enforce its provisions. In general, an intended beneficiary is one to whom the promise of the performance will satisfy the obligation of the promisee to pay money to the beneficiary or the circumstances indicate that the promisee intends to give the beneficiary the benefit of the promised performance. The promisee must have an intention (explicit or implicit) to benefit the third party, or the beneficiary is incidental.

A
436
Q

To recover for invasion of privacy based on the public disclosure of private facts, the plaintiff must show that (i) the defendant gave publicity to a matter concerning the private life of another and (ii) the matter publicized is of a kind that would be highly offensive to a reasonable person and is not of legitimate concern to the public

A
437
Q

In a criminal case, the defendant can only be found guilty if the elements of the crime are proved under the higher standard of beyond a reasonable doubt rather than the lower standard of a preponderance of the evidence that applies for most civil causes of action

A
438
Q

Under the Free Exercise Clause of the First Amendment, a state may enact a neutral law of general applicability.

A
439
Q

A deceased person cannot legally be defamed. The estate of the deceased official cannot maintain an action for defamation because the defamatory statement was made after the official’s death.

A
440
Q

The Commerce Clause reserves for Congress the power to regulate commerce among the states. As a corollary, the “Dormant” Commerce Clause limits the power of the states to legislate in ways that impact interstate commerce. Specifically, state statutes cannot (i) discriminate against out-of-state commerce, (ii) unduly burden interstate commerce, or (iii) regulate extraterritorial (wholly out-of-state) activity. A state or local regulation discriminates against out-of-state commerce if it protects local economic interests at the expense of out-of-state competitors.

A
441
Q

Even though a judge has decided that evidence, such as a confession, is admissible, a party may nevertheless introduce other evidence that is relevant to the weight and credibility of the admitted evidence. Here, the defense seeks to discredit the confession by introducing evidence that it was not given voluntarily.

A
442
Q

In determining whether to set aside a jury verdict, it is not sufficient that the verdict is against the weight of the evidence. Some appellate courts refuse to set aside a jury verdict if there is substantial evidence supporting the verdict. Other courts require only sufficient evidence to sustain a jury verdict. Still others uphold a jury verdict unless there is no evidence to support the verdict. Because there was substantial evidence to support the verdict in this case, the judgment should be affirmed regardless of the requisite level of evidence necessary to support the verdict in the applicable jurisdiction.

A
443
Q

The widow was an intended beneficiary of the agreement between the children. In this case, she was the one to whom the promisee wished to make a gift of the promised performance. An intended beneficiary may sue a promisor to enforce the contract once her rights have vested. The rights of an intended beneficiary vest when the beneficiary (i) detrimentally or materially relies on the rights created; (ii) manifests assent to the contract at one of the party’s request; or (iii) files a lawsuit to enforce the contract. Here, none of these three vesting events occurred before the rescission of the contract. Consequently, the widow cannot enforce the contract.

A
444
Q

Contracts with minors are voidable, but only by the minor.

A
445
Q

To trigger strict or intermediate scrutiny under the Equal Protection Clause of the Fourteenth Amendment, there must be discriminatory intent on the part of the government. The fact that legislation has a disparate effect on people of different races, genders, etc., without intent, is not sufficient to trigger strict or intermediate scrutiny. Instead, proof of discriminatory motive or intent is required.

A
446
Q

Normally a shipment of goods by a seller made in response to an order placed by the buyer constitutes acceptance of the buyer’s offer. Such a shipment does not constitute acceptance, however, if the seller indicates that the shipped goods are made as accommodation. Since the supply company so designated the blades that it sent, the shipment did not constitute acceptance. Consequently, no contract was formed, so there can be no breach

A
447
Q

The political question doctrine applies only to the federal government, not the states

A
448
Q

A seller has the right to cure a defective tender if (i) the time for performance under the contract has not yet lapsed, or (ii) the seller had reasonable grounds to believe the buyer would accept the goods despite the nonconformity

A
449
Q

In determining whether an object is a fixture that can be removed by a seller of real property, the seller’s subjective intent is not controlling. Instead, various factors are examined to ascertain the seller’s objective intent.

A
450
Q

While the declaration gives the developer a general power to amend the declaration with respect to lots owned by the developer, the developer may not use this power in a way that would materially change the character of the development or the burdens on the existing community members unless the declaration fairly apprises purchasers that the power could be used for the kind of change proposed. The change in the type of residences that may be constructed on the developer’s lots likely represents a material change

A
451
Q

If a pleading contains any insufficient defense, or redundant, immaterial, impertinent, or scandalous material, then the court, upon motion or upon its own initiative, may order that such defense or material be stricken. Since the statement in question concerns the value of the automobile, and the complaint only seeks to recover damages for personal injuries, the statement is immaterial.

A
452
Q

Felony murder is an unintended killing proximately caused by and during the commission or attempted commission of an inherently dangerous felony, including a robbery. Most states apply the agency theory when a bystander is killed by a police officer or due to resistance by the victim of the felony. Under this theory, the felon will not be liable for the death of a bystander caused by a felony victim or police officer because neither person is the felon’s agent

A
453
Q

The Section 5 Enabling Clause of the Fourteenth Amendment permits Congress to pass legislation to enforce the equal protection and due process rights guaranteed by the Fourteenth Amendment, as long as there is “congruence and proportionality” between the injury to be prevented or remedied and the means adopted to achieve that end. Congress may override state government action that infringes upon Fourteenth Amendment rights if the “congruence and proportionality” test is satisfied, but it may not regulate wholly private conduct under this amendment.

A
454
Q

There is no federal equal protection clause. Although there is no federal equal protection clause, the Supreme Court has held that the Fifth Amendment Due Process Clause includes the rights guaranteed by the Equal Protection Clause of the Fourteenth Amendment. However, this only means that discrimination by the federal government is subject to the same standard as discrimination by the states—it does not give Congress the power to pass legislation to enforce the equal protection and due process rights guaranteed by the Fourteenth Amendment

A
455
Q

Although the niece did breach the covenant of the right to convey (as well as the covenant of seisin), this breach gives rise to the right to monetary damages. Since the buyer is instead seeking possession of the home, this breach would not aid the buyer’s cause. Answer choice A is incorrect because under the after-acquired property doctrine, if a person (such as the niece) purports to transfer property that she does not own, the subsequent acquisition of the property by the transferor automatically operates to transfer ownership of the property to the transferee. Therefore, application of this doctrine would support the buyer’s claim. Answer choice C is incorrect because under the estoppel by deed doctrine, the niece would be estopped from denying the validity of the deed upon her subsequent acquisition of ownership of the property. This doctrine would therefore also support the buyer’s claim. Answer choice D is incorrect because both the after-acquired property doctrine and the estoppel by deed doctrine apply where the conveyance of property has been made by a general warranty deed. Therefore, the niece’s conveyance of the home by a general warranty deed prior to her actual acquisition of the property would support the buyer’s claim. Note that had the transfer been by a quitclaim deed, these doctrines would not apply.

A
456
Q

A criminal defendant’s right to be represented by an attorney of his own choosing is subject to limitations, including the Sixth Amendment’s guarantee of an effective advocate for a criminal defendant. Answer choice A is incorrect because, although a court must disqualify an attorney from representing co-defendants where there is an actual, present conflict, a court may also disqualify an attorney where there is a serious potential for a conflict of interest. Answer choice B is incorrect because, although a criminal defendant may be permitted to waive a potential conflict of interest, the court is not required to accept such a waiver. Answer choice C is incorrect. The Sixth Amendment, subject to exceptions, protects not only the right to an attorney, but also the right to an attorney of one’s choosing.

A